Sunteți pe pagina 1din 59

2

" Item: 1 of 44
Q.ld: 3170 (
IIPMark <J
Previous
[:>
Next
jf
Lab Values
~
Notes
~
Calculator
3
4
5
6 A 65-year-old man presents w ith right shoulder pain and w eakness after falling on his outstretched hand. The
7 pain is w orse w hen he tries to position his arm above the shoulder level or w hen he pulls or pushes. He also
8 complains of pain w hen he lies on the affected shoulder. He has a 40-pack-year history of cigarette
9 smoking. His father died of multiple m yeloma . His vital signs are w ithin normal limits. Examination show s
10 limitation of mid arc abduction and external rotation that does not improve after lidocaine injection. There is no
11 sw elling, redness or w armth of the joint. Radial pulse is normal. Muscle tone and bulk is w ithin normal
12 limits. W hich of the follow ing w ould be most helpful in confirming the patient's diagnosis?
13
14
15 A. X-ray shoulder [23%]
16 B. Chest X-ray [2%]
17 C. Bone marrow biopsy [1 %]
18
19 .; D. MRI of the shoulder [69%]
20 E . MRI of the cervical spine [2%]
21 F. Aspiration of the joint [1 %]
22
23 G. Bone scan [1 %]
24
25 Explanation: User ld:
26
?.7 This patient is most likely suffering from a rotator cuff tear. This condition presents w ith shoulder pain and
18 w eakness. The pain is aggravated by pushing, pulling, lifting the arm above the head, and lying on the
19 affected shoulder. Rotator cuff tears may occur as the end result of chronic rotator cuff tendonitis or as a
20 result of trauma . The classic presentation of trauma-induced rotator cuff tear involves a fall on an
21 outstretched hand. Limitation of mid arc abduction and external rotation are common findings on exam . The
22 fact that this patient's limitation of movement remains after lidocaine injection provides strong support for a
23 diagnosis of rotator cuff tear. W hile rotator cuff tendinitis may present w ith similar pain and limitation of
24 movement, the symptoms of rotator cuff tendinitis resolve w ith lidocaine injection. An MRI of the shoulder is
25 the test of choice for diagnosing a rotator cuff tear.
26
?.7 (Choice A) An x-ray of the shoulder is an inexpensive w ay of diagnosing fractures, dislocations, and calcific
18 tendonitis. How ever, MRI is superior at evaluating soft tissue structures of the shoulder.
19
20 (Choice B) With this patient's extensive smoking history, a lung cancer involving the brachial plexus could
cause shoulder pain . How ever, brachial plexus damage is not corroborated by this patient's exam.
18
19 (Choice C) Though this patient has a family history of multiple m yeloma, his clinical picture is not consistent
20 w ith this diagnosis. Multiple m yeloma typically presents w ith pain in the spine, ribs, or back. It may be
21 accompanied by hypercalcemia and anemia . Bone marrow biopsy is a poor choice to evaluate this patient's
22 shoulder complaints.
23
24 (Choice E) MRI of the spine w ould help to identify the cause of cervical radiculopathy. This condition may
25 result from disc herniation (osteophytes) and presents w ith arm w eakness, paresthesias, and neck pain .
26
27 (Choice F) Aspiration of synovial fluid is useful for identifying septic and crystal-induced arthritis. The
28 absence of sw elling, w armth and fever make these diagnoses unlikely.
29
(Choice G) Bone scans are used to diagnose osteomyelitis, fractures, and metastatic disease. This patient's
30
history and physical is not consistent w ith one of these diagnoses.
31
32 Educational objective:
33 Rotator cuff tears result from chronic rotator cuff tendonitis and shoulder trauma . Shoulder pain and
34 w eakness w hen lifting the arm above the head is suggestive of rotator cuff pathology. A lidocaine injection
35 ameliorates the pain and w eakness of rotator cuff tendonitis, w hile it does not improve symptoms of a rotator
36 cuff tear. MRI is excellent at visualizing soft tissue structures and is the study of choice for diagnosing rotator
37 cuff tears.
38
39
Time Spent: 4 seconds Copyright UW orld Last updated: [2/ 19/2015]
40
1 ...
Item: 2 of 44
Q. ld : 4581 [
Ill P Mark -<J
Previous
t:>
Ne><t
jf
Lab Values
~
Notes
~
Calculator
3
4
5
6 A 52-year-old man comes to the physician with a long history of joint pain . The patient has pain and stiffness
7 of the small joints of his hands. The symptoms are w orse in the morning and can last several hours. He also
8 has digit sw elling. The patient's hands are shown in the image below:
9
10
11
12
13
14
15
16
17
18
19
20
21
22
23
24
25
26
?.7
18
19
Displayed v;ith penuission fum Spring Healtbcare Ltd. Copyrigb12005 by Current Medicine
I
20
Which of the following is the most likely diagnosis?
21
22
23 A. Crystalline arthritis [3%1
24 B. Dermatomyositis [8%1
25
C. Enteropathic arthritis [1 %1
26
?.7 D. Neuropathic arthropathy [0% 1
18 E. Osteoarthritis [4%1
19
.; F. Psoriatic arthritis [73% 1
20
21 G. Rheumatoid arthritis [1 0%1
22 H. Sarcoidosis [1 %1
23
24
25 Explanation: User ld:
26
?.7
18 Clinical features of psoriatic arthritis
19
20
21 DIP joints
22 Asymmetric oligoarthritis
23 Arthritis Symmetric polyarthritis, similar to rheumatoid arthritis
24 Arthritis mutilans (deforming and destructive arthritis)
25 Spondyloarthritides (sacroiliitis and spondylitis)
26
?.7
18 Enthesitis (inflammation at tendon insertion site to bone)
19 Soft t issue
Dactylitis ("sausage digits") of toe or finger
20 & nail
involvement Nail pitting & onycholysis
21
22
Swelling of the hands or feet with pitting edema
23
24 Arthritis precedes skin disease in 15% of patients
25 Skin lesions Skin lesions are present but not yet diagnosed in 15%
26
of patients
?.7
18 @UWo~d
19
20 The clinical history and image provided are most consistent with psoriatic arthritis. Psoriatic arthritis occurs
21 in 5%-30% of patients who have psoriasis. Psoriatic arthritis can present with many different patterns, but the
22 classic presentation involves the distal interphalangeal (DIP) joints. Morning stiffness is present, as it is
23 in all inflammatory arthritides. Deformity of involved joints, dactylitis ("sausage digit," a diffusely sw ollen
24 finger), and nail involvement are common . This patient show s all of the classic signs: All of the fingers
25 (particularly the left index finger) are sw ollen, consistent with dactylitis; many of the nails show pitting and
26 onycholysis (separation of nail bed); and the DIP joints are prominently involved. Furthermore, w ell-
?.7 demarcated red plaques with silvery scaling- the classic lesions of psoriasis- are seen on the dorsum of
18 each hand. Current treatment options for psoriatic arthritis include nonsteroidal anti-inflammatory agents,
19 methotrexate, and anti-tumor necrosis factor agents.
20
21 (Choice A) Nail changes do not accompany crystalline arthritis (gout and pseudogout). Cutaneous deposits
22 of monosodium urate occur in chronic gout. These lesions, known as tophi, most commonly occur over
23 joints and the helix of the ear, and they may ulcerate.

I
24
25 (Choice B) Dermatomyositis can present with Gottron's papules (violaceous plaques, slightly scaly)
26 overlying the metacarpophalangeal (MCP) joints, which at times can look similar to psoriasis. How ever,
?.7 dermatomyositis is not associated with dactylitis or nail changes (onycholysis or nail pitting).
18
19 (Choice C) Enteropathic arthritis occurs in 10%-20% of patients with Crohn disease and ulcerative colitis. It
20 most often affects the low er extremities and sacroiliac joints and tends to w ax and w ane with the symptoms
21 of bow el disease. These patients have prominent gastrointestinal symptoms and other extraintestinal
22 manifestations of inflammatory bow el disease.

18 (Choice D) Neuropathic arthropathy (Charcot joint) most commonly affects the low er limbs. Most cases
19 occur with diabetes, but syphilis and alcoholism are other causes. Peripheral neuropathy is believed to result
20 in decreased proprioception, which leads to frequent trauma and eventual destruction of the joint.
21
22 (Choice E) Osteoarthritis often affects the DIP joints. How ever, the pain of osteoarthritis is worse with
23 activity and improves with rest (the opposite of this patient's symptoms). Osteoarthritis is noninflammatory,
24 and dactylitis should not occur.
25
26 (Choice G) Rheumatoid arthritis also presents with morning stiffness. The MCP and proximal
27 interphalangeal joints are prominently involved, whereas the DIP joints classically are not. Dactylitis and nail
28 involvement are also not expected in rheumatoid arthritis.
29
(Choice H) Polyarthritis affecting the ankles and knees sometimes occurs in sarcoidosis. Sarcoidosis
30
causes protean cutaneous manifestations, the most common of which is erythema nodosum. These
31
patients also often have cough, chest pain, and dyspnea .
32
33 Educational objective:
34 Psoriatic arthritis occurs in 5%-30% of patients with psoriasis. The classic presentation involves the distal
35 interphalangeal joints. Morning stiffness, deformity, dactylitis \'sausage digit"), and nail involvement are
36 common . Current treatment options for psoriatic arthritis include nonsteroidal anti-inflammatory agents,
37 methotrexate, and anti-tumor necrosis factor agents.
38
39
Time Spent: 6 seconds Copyright UW orld l ast updated: [1 0/30/20141
40
Media Exhibit
Media Exhibit
Media Exhibit

Erythema nodosum
1
2
" Item: 3 of 44
Q.ld : 4743 [
Il l!'' Mark <:]
Previous
t>
Next
il
Lab Values
~
Notes
~
Calculator

4
5
6 A 51-year-old man complains of difficulty w alking and mild right-sided foot pain for the past several w eeks.
7 The patient's medical history is significant for type 1 diabetes mellitus, hypertension, and
8 hypercholesterolemia. Physical examination show s a significantly deformed right foot and a mildly deformed
9 left foot. X-rays of the right toot suggest effusions in several of the tarsometatarsal joints, large osteophytes,
10 and several extra-articular bone fragments. Which of the following is the most likely cause of this patient's
11 complaints?
12
13
14 A. Avascular necrosis [7%]
15 B. Bacterial infection [3%]
16 C. Bone demineralization [18%]
17
18 D. Hyperuricemia [12%]
19 ~ E. Nerve damage [45%]
20 F. Rheumatoid arthritis [1 6%]
21
22
23 Explanation: User ld:
24
25
26 Charcot joint (neurogenic arthropathy)
?.7
18 Vitamin 812 deficiency
19
Diabetes
20
21 Associated Peripheral nerve damage
22 conditions Spinal cord injury
23 Syringomyelia
24
25 Tabes dorsalis (tertiary syphilis)
26
?,7 Deformed joints
18
Lacking/decreased sensation (proprioception, pain,
19
20 temperature) with loss of neurologic input
21 Clinical Arthritis or arthropathy
22 manifestations
Mild pain
23
24 Fractures (may be unsuspected by patient)
25 Degenerative joint disease & loose bodies on joint imaging
26
?.7
18 Treat underlying condition
19
Mechanical devices (assist in weight bearing, decrease
20 Management
further trauma)
21
22 X-rays (if trauma is present)
23
24 @USMLEWorld, LLC

25
This patient has neurogenic arthropathy (Charcot joint) due to diabetes. Pathogenesis involves decreased
26
?,7 proprioception, pain, and temperature perception, which can occur due to diabetes, peripheral nerve
18 damage, syringomyelia, spinal cord injury, vitamin B 12 deficiency, or tabes dorsalis. As normal neurologic
input is lost, patients unknowingly traumatize their w eight-bearing joints. This causes secondary
19
degenerative joint disease, joint deformation, and functional limitation. X-rays will reveal loss of cartilage,
20
osteophyte development, and loose bodies. In diabetes, vasculopathy also contributes to disease
21
pathogenesis. Associated pain is typically mild. Management of a Charcot joint involves treating the
22
23 underlying disease and providing mechanical devices (eg, special shoes) to assist in w eight bearing and
decrease further trauma.
24
25
(Choice A) Avascular necrosis refers to bone destruction due to poor vascular supply. It is a complication of
26
?.7 corticosteroid use, trauma, systemic lupus erythematosus, sickle cell disease, and a number of other
processes. It generally affects the proximal and distal femur with pain as the main symptom.
18
19 (Choice B) Bacterial infection is an important cause of acute monoarticular arthritis (septic joint). ~ is
20 exquisitely painful and rapidly destructive. Fever may be present. An x-ray will reveal joint destruction but not
21 osteophytes or loose bodies. A septic joint is diagnosed by joint fluid aspiration.
18
19 (Choice C) Bone demineralization is the hallmark of osteoporosis. This disease predisposes patients to
20 pathologic fractures, particularly of the hip and vertebrae. How ever, it does not cause joint destruction.
21
22 (Choice D) Gout results from uric acid deposition within joints. It is exquisitely painful and generally affects
23 the great toes, ankles, wrists, and elbows. X-rays of joints affected by chronic gout reveal punched-out
24 erosions with an overhanging rim of cortical bone known as a "rat bite" lesion.
25
26 (Choice F) Rheumatoid arthritis is a common cause of polyarticular inflammatory arthritis. It typically affects
27 the hands, w rists, feet, and ankles, generally in a symmetric fashion . X-rays of patients with long-standing,
28 poorly controlled disease will reveal periarticular osteoporosis, joint erosions, and joint space narrowing.
29
Educational objective:
30
Neurogenic arthropathy (Charcot joint) is a complication of neuropathy and repeated joint trauma. It affects
31
w eight-bearing joints and manifests with functional limitation, deformity, and degenerative joint disease and
32
loose bodies on joint imaging.
33
34
35 References:
36 1. Charcot neuroarthropathy: An often overlooked complication of diabetes.
37
38
39
Time Spent: 3 seconds Copyright UWorld Last updated: [1 2121 /2014]
40
- ~-- --- -~ - -- ----- - - - - - - -
1
2
" Item: 4 of 44
Q.ld : 4587 [
Il l!'' Mark <:]
Previous
t>
Next
il
Lab Values
~
Notes
~
Calculator
3

5
6 A 24-year-old w oman comes to the physician due to w eakness, facial rash, low -grade fever, and joint pain.
7 She has pain and sw elling of the hand joints and pain in the knees. Her mother has rheumatoid arthritis. The
8 patient's blood pressure is 145/90 mm Hg and her heart rate is 90/min. Her hematocrit is 40% and
9 erythrocyte sedimentation rate is 63 mm/h. Urinalysis is 2+ for protein . Which of the following is the best
10 statement about this patient's joint symptoms?
11
12
A Bone resorption of the distal phalanges may result [5%]
13
14 B. Cartilage degradation and muscle atrophy cause deformity [11 %]
15 C. Excessive bony growth is characteristic [5%]
16
~ D. Permanent deformity is uncommon [58%]
17
18 v E. Subluxation and tendon damage cause permanent deformity [21 %]
19
20
Explanation: User ld:
21
22
23 Manifestations of systemic lupus erythematosus
24
25
26 Constitutional: Fever, fatigue & weight loss
?.7 Symmetric, migratory arthritis
18
19 Skin: Butterfly rash & photosensitivity
Clinical
20 sym ptoms Serositis: Pleurisy, pericarditis & peritonitis
21
22 Thromboembolic events (due to vasculitis &
23 antiphospholipid antibodies)
24 Neurologic: Cognitive dysfunction & seizures
25
26
?,7 Hemolytic anemia, thrombocytopenia & leukopenia
18
Hypocomplementemia (C3 & C4)
19
20 Laboratory Antibodies:
21 fin dings o Anti-ANA (sensitive)
22
23 o Anti-dsDNA & anti-SM (specific)
24
Renal involvement: Proteinuria & elevated creatinine
25
26 @UWortd
?.7
18
This patient has features suggestive of systemic lupus erythematosus (SLE), including fever, facial rash,
19
arthralgias, anemia, elevated inflammatory markers, and evidence of nephropathy. Young w omen and
20
individuals with a family history of autoimmune disease are at greatest risk for SLE. Arthritis affects 90% of
21
patients with SLE and most commonly occurs in the hands or knees. It is a migratory arthritis with pain that
22
is often disproportionate to objective physical findings. SLE arthritis has a low er incidence of erosion, synovial
23
abnormality, and permanent joint deformity than rheumatoid arthritis and is considered a non-deforming
24
arthritis.
25
26 Management of joint symptoms in patients with SLE is usually straightforward and often dependent on the
?,7 severity of systemic manifestations. Most patients will respond to conservative treatment with nonsteroidal
18 anti-inflammatory drugs. Low -dose glucocorticoids or antimalarials (eg, hydroxychloroquine) can be added for
19 more severe joint symptoms. This patient will likely require systemic therapy due to evidence of lupus
20 nephritis.
?j
18 (Choice A) Distal phalangeal resorption is characteristic of the arthritis mutilans variant of psoriatic arthritis,
19 leading to radiologic "pencil-in-cup" deformities. Bone resorption does not occur in SLE.
20
21 (Choice B) Cartilage degradation is the underlying pathologic mechanism of osteoarthritis (OA). Patients
22 with OA may develop muscle atrophy from limitations of joint movement.
23
24 (Choice C) Osteophytes are bony growths often seen around the joints of patients with OA. This patient's
25 age, family history, and systemic illness are more suggestive of SLE.
26
27 (Choice E) Subluxation of the cervical vertebrae and tendon damage in the hands may occur in rheumatoid
28 arthritis.
29
Educational objective:
30
Systemic lupus erythematosus (SLE} is an autoimmune disease that is most commonly diagnosed in young
31
w omen. More than 90% of patients with SLE have joint involvement, most often affecting the hands. Joint
32
pain may be severe but SLE induced arthritis is usually non-deforming.
33
34
35 References:
36 1. How to treat refractory arthritis in lupus?
37
38
39
Time Spent: 4 seconds Copyright UW orld Last updated: [2/1 0/2015] v
40
Media Exhibit
Media Exhibit
1
2
" Item: 5 of 44
Q. ld : 4110 [
Il l!''Mark <:]
Previous
t>
Next
il
Lab Values
~
Notes
~
Calculator
3
. 4

6 A 70-year-old woman comes to the physician due to joint pains in both hands. The patient has no other
7 symptoms. Her other medical problems include obesity and gastroesophageal reflux disease. She does not
8 use tobacco, alcohol, or illicit drugs. Family history is not significant. Her medications include omeprazole
9 and acetaminophen. Her vital signs are w ithin normal limits. X-ray of the joints is show n below .
10
11
12
13
14
15
16
17
18
19
20
21
22
23
24
25
26
?.7
18
19
20
21
22
23
24
25 W hich of the follow ing is the most likely diagnosis?
26
?,7 A. Gouty arthritis (1 %1
18 B. Hemochromatosis [0%1
19
20 ., C. Osteoarthritis [78%1
21 D. Reactive arthritis [2%1
22 E. Rheumatoid arthritis [18%1
23
24 F. Systemic lupus erythematosus [0%1
25
26 Explanation: User ld:
?.7
18
19
20
21
22
23
24
25
26
?,7
18
19
20
21
22
23
24
25
26
?.7
18
19
20
21
This patient's hand x-ray is classic for degenerative joint disease or osteoarthritis; the bones are also
18 osteoporotic. Compared to a normal hand x-ray, there is diffuse degenerative joint space narrow ing, w hich
19 involves all the distal interphalangeal (DIP) joints and, to a lesser extent, the proximal interphalangeal (PIP)
20 and metacarpophalangeal (MCP) joints. Subchondral sclerosis and some osteophytes are also seen .
21
22 (Choice A) Most patients (80%) w ith gout present initially w ith monoarthritis but only 20% have polyarticular
23 involvement.
24
25 (Choice B) The characteristic radiographic findings of hemochromatosis-associated arthropathy include
26 squared-off bone ends and hook-like osteophytes in the second and third MCP joints.
27
28 (Choice D) Reactive arthritis typically follow s an infectious illness and is manifested by asymmetric
29 oligoarthritis.
30
31 (Choice E) Rheumatoid arthritis classically spares the DIP joints and involves the MCP and PIP joints.
32
(Choice F) Systemic lupus erythematosus can also affect the hand joints. However, it is unlikely to present
33
at such an advanced age and w ithout other systemic manifestations.
34
35 Educational objective:
36 Osteoarthritis predominantly involves the distal interphalangeal joints. The major radiographic features include
37 joint space narrow ing, subchondral sclerosis, osteophytes, and subchondral cysts.
38
39
Time Spent: 6 seconds Copyright UWorld Last updated: [11 / 10/20141
40
Media Exhibit
Media Exhibit
1
2
" Item: 6 of 44
Q.ld: 3574 (
IIPMark <J
Previous
[:>
Next
jf
Lab Values
~
Notes
~
Calculator
3
4
5
A 33-year-old tennis player comes to you w ith a complaint of pain in his right shoulder. He says that the pain
7 is absent at rest but present w hen he lifts his arm over his head. The pain is compromising his play. On
8 examination, active motion at right shoulder is limited due to pain. Pain is most severe on passive internal
9 rotation and flexion at the right shoulder. No atrophy of the shoulder muscle is seen. W hich of the follow ing is
10 the most likely diagnosis in this patient?
11
12
- A. Tear of long head of bicep tendon [12%]
13
14 B. Tennis elbow [4%]
15 ~ C. Subacromial bursitis [74%]
16
D. Anterior dislocation of shoulder [7%]
17
18 ~ E . Axillary nerve palsy [3%]
19
20
Explanation: User ld:
21
22 The patient described is suffering from subacromial bursitis. The subacromial bursa lies between the
23 acromion and the tendon of the supraspinatus muscle. Inflammation of the subacromial bursa typically
18 occurs in the setting of chronic microtrauma to the supraspinatus tendon caused by overhead w ork and
19 repeated overhead movements of the arm during w ork or sporting activities. The tendon is traumatized by
20 compression between the acromion and the humeral head, and its vascular supply may be temporarily
21 compromised during such episodes of compression. The physical examination in subacromial bursitis
22 reveals pain w ith active range of motion of the shoulder, tenderness in the shoulder w hen the arm is internally
23 rotated and forward flexed at the shoulder (Neer impingement sign) and no signs of deltoid atrophy.
24
25 (Choices A & D) A tear of the proximal end of the long head of bicep tendon is associated w ith rotator cuff
26 injury and typically occurs during forceful flexion of the arm. Patients present w ith a prominent bulge in the
27 midportion of the upper arm. Axillary nerve injury may occur as a result of anterior dislocation of the humerus
28 out of the glenoid fossa .
29
30 (Choice B) Tennis elbow causes chronic nagging pain over the lateral epicondyle (lateral epicondylitis).
31
(Choice E) The axillary nerve supplies the deltoid and the teres minor w ith motor innervation. Palsy of this
32
nerve w ould result in paralysis and ultimately denervation atrophy of these muscles.
33
34 Educational objective:
35 Subacromial bursitis is the result of repetitive overhead motions. Patients complain of pain w ith active range
36 of motion of the shoulder, and passive internal rotation and forward flexion at the shoulder also elicits
37 tenderness.
38
39
Time Spent: 4 seconds Copyright UW orld Last updated: [2/ 19/2015]
40
1
2
" Item: 7 of 44
Q. ld : 4582 (
I IP Mark <J
Previous
[:>
Next
jf
Lab Values
~
Notes
~
Calculator
3
4
5
6 A 67 -year-old male hospitalized after elective hernia repair complains of severe right knee pain . Physical
examination reveals redness and swelling of the right knee with limited motion due to pain. His temperature is
8 39.9 C (102 F), blood pressure is 160/ 110 mm Hg, pulse is 80/min, and respirations are 16/min. Synovial
9 fluid analysis reveals the following findings:
10 3
11 WBC count 30,000mm
12 Neutrophils 90%
13 Crystals rhomboid-shaped, positively birefringent
14 Gram stain negative
15
16 Which of the following is most likely associated with this patient's current condition?
17
18 A. Tophi [1 7%1
19
B. Transient bacteremia [8%1
20
21 .; C. Chondrocalcinosis [68%1
22 D. Rheumatoid factor [2%1
23
E. Heberden nodes [3%1
24
25 r.... F. Chlamydia! infection [2%1
26
?.7
Explanation: User ld:
18
19 Pseudogout is a form of acute arthritis induced by the release of calcium pyrophosphate dehydrate (CPPD}
20 crystals from sites of chondrocalcinosis (calcification of articular cartilage} into the joint space. It often occurs
21 in the setting of recent surgery or medical illness. Patients present with acute pain, swelling, redness and
22 limited motion of the involved joint(s}, with the knee being most commonly affected. Fever, leukocy1osis and a
23 left shift may occur in pseudogout, although the presence of these findings is not required for diagnosis. The
24 crystals of CPPD can be difficult to detect on synovial fluid analysis, but the identification of rhomboid-shaped,
25 positively birefringent crystals is diagnostic.
26
?.7 (Choice A) Tophi are collections of urate crystals which form firm, yellowish nodules at the sites of involved
18 joints in patients with chronic gout. The presentation of gout and pseudogout can be similar, but gout is
19 distinguished by needle-shaped, negatively birefringent crystals.
20
21 (Choice B) Transient bacteremia could lead to septic arthritis causing acute pain and swelling of the knee in
?.? the setting of fever and leukocy1osis. How ever, the negative gram stain and presence of rhomboid-shaped,
18 positively birefringent crystals in this case make pseudogout the more likely explanation. S. aureus is the most
19 common cause of septic arthritis.
20
21 (Choice D) Positive rheumatoid factor (RF} is characteristic of rheumatoid arthritis (RA}, but finding a positive
22 RF does not necessarily make the diagnosis of RA as it can occur in many other conditions. RA presents
23 with gradual onset symmetric arthritis; fever may be present. This patient's acute onset of pain, recent
24 surgery and crystals in the synovial fluid are more consistent with pseudogout.
25
26 (Choice E) Heberden nodes occur in the DIP joints in osteoarthritis and form from calcified cartilaginous
27 spurs. Osteoarthritis may predispose to pseudogout, but osteoarthritis alone cannot account for the acute
28 onset, inflammation, fever and synovial fluid analysis in this case.
29
(Choice F) Chlamydia! infection can result in reactive arthritis. The triad of urethritis, conjunctivitis and
30
arthritis compose the classic presentation. Lower extremity oligoarthritis and enthesopathy (inflammation at
31
tendon and ligament insertions} account for musculoskeletal complaints.
32
33 Educational objective:
34 Acute inflammatory arthritis caused by CPPD crystals is called pseudogout. It often occurs in the setting of
35 recent surgery or medical illness. Pseudogout is diagnosed by the presence of rhomboid-shaped, positively
36 birefringent crystals on joint aspiration microscopy and radiographic evidence of chondrocalcinosis (calcified
37 articular cartilage}.
38
39
Time Spent: 4 seconds Copyright UWorld Last updated: [2/ 19/20151
40
1
2
" Item: 8 of 44
Q.ld : 3317 [
Il l!'' Mark <:]
Previous
t>
Next
il
Lab Values
~
Notes
~
Calculator
3
4
5
6 A 70-year-old man comes to the physician for pain and stiffness of his neck, shoulders, and hips for the last 3
7 months. His stiffness is w orse in the morning and lasts 1-2 hours. He also complains of general malaise and
a recent w eight loss of 3.17 kg (7 lb). The patient has no headache, scalp tenderness, visual symptoms, or
9 jaw claudication. Examination show s no overt synovitis at the joints, with normal passive range of motion.
10 The arteries of the scalp, neck, and ex1remities are normal on palpation without tenderness. Laboratory
11 results are as follow s:
12 Leukocytes 7500/~L
13 Hematocrit 31%
14 Platelets 450,000/~L
15 Erythrocyte sedimentation rate 85 mm/h
16 Thyroid-stimulating hormone 1.8 ~U/mL
17 Creatine kinase 33 U/L
18
19
Which of the following is the most appropriate nex1 step in management of this patient?
20
21
22 .~ A Measure antinuclear antibodies and rheumatoid factor levels [32%]
23 B. Recommend low-impact aerobic exercise [7%]
24
C. Temporal artery biopsy [4%]
25
26 .; D. Treatment with low -dose prednisone [40%]
?.7 E. Treatment with nonsteroidal anti-inflammatory drug [1 7%]
18
19
20
21
Explanation: User ld: I
22 Polymyalgia rheumatica
23
24
25 Clinical
26 Findings
features
?,7
18
19 Age >50
20 Bilateral pain & morning stiffness >1 month
21 Involvement of 2 of following:
22 Symptoms o Neck or torso
23 o Shoulders or proximal arms
24
o Proximal thigh or hip
25
26
o Constitutional (fever, malaise, weight loss)
?.7
18 Physical
19 Decreased active ROM in shoulders, neck & hips
examination
20
21
22 ESR >40 mm/h, sometimes >100 mm/h
23 Laboratory Elevated CRP
24
studies Normocytic anemia possible
25
26 - 20% can have normal studies
?,7
18 Treatment Response to glucocorticoids
19
20 CRP= C-reactive protein; ESR = erythrocyte sedimentation rate; ROM = range of motion.
21
@UWOI1d
22
23
24 This patient has the following characteristic features of polymyalgia rheumatic a (PMR):
25 Age >50
26 Subacute-to-chronic (>1 month) pain in the shoulder and hip girdles
?.7 Morning stiffness lasting > 1 hour
18 Constitutional symptoms
19 Elevated erythrocyte sedimentation rate >40 mm/h
20 No other apparent explanation for symptoms
21
22
23 In PMR, the physical examination is frequently unremarkable with patients having no focal tenderness or pain
24 with active or passive range of motion. Signs of inflammation in the joints are absent. When asked to identify
25 the location of their pain, patients typically indicate the soft tissues and not the joints.
26
?,7 Low -dose glucocorticoids are the treatment of choice for PMR (eg, prednisone 10-20 mg daily). Rapid and
thorough relief of symptoms is expected, and failure to improve rapidly on prednisone should call the
18
diagnosis into question. PMR is frequently associated with giant cell arteritis (GCA), also known as temporal
19
20 arteritis. Symptoms of GCA include headache, jaw claudication, vision loss, or tenderness over the temporal
artery. If GCA is suspected, patients should be considered for an expedited temporal artery biopsy (Choice
21
C) and receive significantly higher doses of glucocorticoids (eg, prednisone 40-60 mg or higher daily).
22
23 (Choice A) Antinuclear antibody and rheumatoid factor are screening tests for systemic lupus erythematosus
24
and rheumatoid arthritis, respectively, and are frequently positive in a number of other inflammatory and
25 autoimmune conditions. These tests have poor specificity in the absence of characteristic symptoms (eg, I
18 malar rash, metacarpophalangeal synovitis) and w ould be unlikely to change management in this patient.
19
20 (Choice B) Fibromyalgia (FM) most commonly presents in young to middle-aged women with widespread
21 pain, fatigue, and cognitive/mood disturbances. Patients have point muscle tenderness in areas such as the
22 mid trapezius, lateral epicondyle, costochondral junction in the chest, and greater trochanter. Regular aerobic
23 exercise and good sleep hygiene are recommended for FM. How ever, constitutional symptoms (eg, w eight
24 loss) or elevated erythrocyte sedimentation rate are not present in FM.
25
26 (Choice E) Nonsteroidal anti-inflammatory drugs are occasionally useful in PMR for mild breakthrough
27 discomfort or minor relapse of symptoms while tapering glucocorticoids. How ever, they are much less
28 effective than glucocorticoids for initial management and are not recommended .
29
Educational objective:
30
Polymyalgia rheumatic a (PMR) affects patients age >50 and is characterized by pain and stiffness in the
31
neck, shoulders, and pelvic girdle, along with an elevated erythrocyte sedimentation rate. The treatment of
32
choice for uncomplicated PMR is low-dose prednisone, which results in rapid relief of symptoms.
33
34
35 References:
36 1. Recognition and management of polymyalgia rheumatica and giant cell arteritis.
37
38
39
Time Spent: 5 seconds Copyright UW orld Last updated: [1 2/112014]
40
1
2
" Item: 9 of 44
Q. ld : 4574 [
Il l!'' Mark <:]
Previous
t>
Next
il
Lab Values
~
Notes
~
Calculator
3
4
5
6 A 52-year-old w oman complains of difficulty gripping her cup of coffee in the morning. She likes to solve
7 crossw ord puzzles but is unable to hold a pen properly due to stiffness in her right hand. Her symptoms
8 gradually improve during the day and she is "fully functional" by late afternoon. Examination show s nontender
nodules near the elbow s over the back of her forearm. The erythrocyte sedimentation rate is 45 mm/hr.
10 W hich of the following w ould most likely be involved during the course of this patient's disease?
11
12
~ A Cervical spine [51%]
13
14 B. Lumbar spine [9%]
15 C. Sacral spine [2%]
16
D. Sacroiliac joints [33%]
17
18 E. Thoracic spine [4%]
19
20
Explanation: User ld:
21
22
23
24 Clinical features of rheumatoid arthritis
25
26 Symptoms
?.7
18 Insidious onset, multiple joint pain, stiffness & swelling
19 Morning stiffness lasting hours, improves with activity
20
21 Small joints (eg, PIP, MCP, MTP) commonly involved
22 Monoarthritis (eg, knees, elbows) can also occur later
23
24 Sp ares the DIP joint, unlike osteoarthritis
Clinical
25 presentation Examination
26
?,7
Affected joints are tender to the touch, swollen, with limited range
18
of motion
19 Tenosynovitis of the palms ~ "trigger finger"
20 Rheumatoid nodules (especially on elbows)
21
22 Cervical joint involvement can lead to spine subluxation ~ spinal
23 cord compression
24
25
26 Positive anti-CCP antibodies (diagnostic testing)
Labo ratory/
?.7 High lgM rheumatoid factor
imaging
18
studies High C-reactive protein & ESR correlate with disease activity
19 X-ray: Soft-tissue swelling, joint space narrowing & bony erosions
20
21 Anti-CCP =anti-cyclic citrullinated peptide; DIP = distal interphalangeal joints;
22 ESR =erythrocyte sedimentation rate; lgM = Immunoglobulin M; MCP = metacarpophalangeal;
23 MTP = metatarsophalangeal.
24 UWor1d
25
26 This patient's presentation (morning stiffness, metacarpophalangeal j oint involvement, nontender nodules on
?,7 the elbow, high erythrocyte sedimentation rate) suggests rheumatoid arthritis (RA). RA is a chronic systemic
18 inflammatory disorder that involves synovial inflammation leading to joint damage. W omen are affected 2-3
19 times more frequently than men, with a peak incidence at age 50-75.
20
21 RA primarily involves peripheral joints and most commonly affects the metacarpophalangeal (MCP) and
22 proximal interphalangeal (PIP) joints of the fingers, metatarsophalangeal (MTP) joints of the toes, and the wrist
23 joints in the early phases. How ever, the distal interphalangeal (DIP) joints are not involved. The disease
24 eventually can involve larger joints such as the knees, elbow s, ankles, and shoulders. Symptoms include
25 pain, swelling, and morning stiffness that improve with increased movement and during the course of the day.
26 Reduced grip strength can occur occasionally and is a sensitive sign of early disease.
?.7
18 Rheumatoid nodules can occur in up to 30%-40% of patients and present as firm and nontender
19 subcutaneous nodules usually close to pressure points (eg, elbow s). RA most commonly affects the cervical
spine joints in the axial skeleton and can lead to cervical spine subluxation, which can also cause spinal cord
:?9
compression. Patients with cervical spine involvement often complain of neck pain, stiffness, and radicular
18
pain in the upper extremity. Subluxation with spinal cord compression can present with hyperreflexia or
19
upgoing toes on Babinski testing.
20
21 (Choices B, C, 0, and E) RA does not affect the thoracic, lumbar, or sacral spine or sacroiliac joints. The
22 thoracic spine has great stability and is rarely affected by arthritis, disc herniation, or other spinal disorders.
23 The lumbar spine is the most common site of disc herniation and spinal stenosis. The sacroiliac joints are
24 commonly affected in seronegative spondyloarthropathies (eg, ankylosing spondylitis, reactive arthritis, and
25 psoriatic arthritis).
26
27 Educational objective:
28 Rheumatoid arthritis (RA) presents as progressive and symmetrical involvement of the peripheral joints. The
29 common sites of early involvement include metacarpophalangeal and proximal interphalangeal joints of the
30 fingers, metatarsophalangeal joints of the toes, and the wrist joints. RA most commonly affects the cervical
31 spine joints in the axial skeleton and can cause spinal subluxation and spinal cord compression .
32
33 References:
34
35 1. Rheumatoid arthritis.
36 2. Rheumatoid arthritis of the cervical spine--clinical considerations.
37
38
39
Time Spent: 3 seconds Copyright UW orld Last updated: [ 1/9/2015]
40
-----------------------------------
1 ...
2 Item: 10 of 44
Q.ld: 9906 [
III P Mark -<J
Previous
t:>
Ne><t
jf
Lab Values
~
Notes
~
Calculator
3
4
5
6 A 28-year-old w oman comes to the clinician with pain and numbness of both hands over the last 6 months.
7 Cold temperatures seem to precipitate these episodes. Medical history includes exercise-induced asthma,
8 lower back pain, and hypothyroidism . The patient takes levothyroxine daily. She does not use alcohol or illicit
9 drugs but has a 10-pack-year smoking history. l aboratory studies show normal complete blood count and
chemistry panel. TSH is 4.71-JU/ml. Antinuclear antibody screen is negative. Physical examination findings
11 are shown in the image below.
12
13
14
15
16
17
18
19
20
21
22
23
24
25
26
?.7
18
19
20
21
22
23
24
25
26
?.7
18
19
20
I
21 In addition to smoking cessation, which of the following is the best initial therapy for this patient?
22
23 ..; A. Amlodipine [7 4%1
24 B. Clonidine [11 %1
25
26 C. Increase in levothyroxine dose [6%1
?.7 D. lisinopril [2% 1
18 ~ E. Metoprolol [7%1
19
20
21 Explanation: User ld:
22
23
24 Secondary Raynaud
25
Primary Raynaud phenomenon
pheno menon
26
?.7
18 Connective tissue diseases
19 (eg, systemic lupus
20 erythematosus, scleroderma)
21
22 Occlusive vascular conditions
23 (eg, Buerger's disease)
24
25 Etiology Sympathomimetic drug use
No underlying cause
26 (eg, ephedrine, epinephrine)
?.7
18 Use of vibrating tools (eg,
19 jackhammer, power drill)
20 Hyperviscosity syndromes
21
22 (eg, cryoglobulinemia,
23 paraproteinemias)
24
25
Usually men age >40
26
?.7
Usually women age <30 Asymmetrical attacks
18 Possible prevalence in multiple Presence of systemic
19 family members symptoms or symptoms of
20
21 Symmetrical episodic attacks another disease
22 Clinical No tissue injury or digital pitting Painful episodes with tissue
23 features injury or digital ulcers
24 Negative nailfold capillary
25 examination Abnormal nailfold capillary
26 Negative antinuclear examination
?.7 antibodies, erythrocyte Abnormal laboratory studies
18 sedimentation rate for vascular or autoimmune
19
disease
20
21
22 Evaluation & treatment for
23 underlying etiology

I
24 No further evaluation
25 Calcium channel blockers for
26 Treatment/ Avoidance of aggravating persistent symptoms; aspirin
?.7 evaluati on factors added in patients with risk for
18 Calcium channel blockers for digital ulceration
19 persistent symptoms Chemical or surgical
20
sympathectomy for severe
21
22
ischemia
23 USMLEWorld, UC
24
25 Clinical features suggest that this patient has primary Raynaud phenomenon (RP), an increased vascular
26 response to cold temperature or emotional stress without an underlying cause. Primary RP occurs usually in
?.7 w omen age <30 with symptoms of symmetrical episodic attacks and without evidence of peripheral vascular
18 disease, connective tissue disease, tissue injury, or abnormal nailfold capillary examination. Abnormalities
19 such as dilated or dropout vessels in the nailfold capillaries are predictive for future development or the
20 presence of a connective tissue disease. Episodes can also involve tingling, numbness, and pain with skin
21 color changes of pallor (white attack) or cyanosis (blue attack). l aboratory studies (eg, antinuclear
22 antibodies, erythrocyte sedimentation rate) are usually negative.
23
24 Primary RP does not require further evaluation. Treatment mainly involves avoiding aggravating factors
18 (ie, cold stimuli, emotional stress). Dihydropyridine calcium channel blockers (eg, nifedipine, amlodipine)
19 are the drugs of choice for continuing symptoms.
20
(Choices B, D, and E) Clonidine, angiotensin II converting enzyme inhibitors, and beta blockers are not
21
helpful for treatment of RP. In fact, some studies have shown that nonselective beta blockers may w orsen
22
the condition.
23
24 (Choice C) Although hypothyroidism can lead to generalized cold intolerance, sharply demarcated digit color
25 changes are not typical of cold-induced vasospasm caused by hypothyroidism .
26
27 Educational objective:
28 Primary Raynaud phenomenon is an increased vascular response to cold temperature or emotional stress. It
29 is usually found in women age <30 with symptoms of symmetrical episodic attacks without evidence of
30 peripheral vascular disease, tissue injury, or abnormal nailfold capillary examination. Treatment involves
31 mainly calcium channel blockers (eg, nifedipine, amlodipine) and avoiding aggravating factors.
32
33 References:
34
35 1. Current medical and surgical management of Raynaud's syndrome.
36 2. The association of Raynaud syndrome with ?-blockers: a meta-analysis.
37
38
39
Time Spent 8 seconds Copyright UW orld l ast updated: [2116/20151
40
1
2
" Item: 11 of 44
Q. ld: 4590 [
IIPMark <J
Previous
[:>
Next
jf
Lab Values
~
Notes
~
Calculator
3
4
5
6 A 24-year-old w oman comes to the physician complaining of a skin rash and pain in her w rists, ankles and
7 elbow s over the past 4 days. She has also had a fever and sw eats, but denies headache, nausea or
8 vomiting . She has no significant past medical history and does not take any medications. She recently took a
9 vacation w ith her new boyfriend. She denies any previous sexually transmitted diseases but acknow ledges
10 having unprotected sex w ith her new boyfriend. She does not use tobacco, alcohol or illicit drugs. Her
temperature is 38.5 C (101 .3 F) and her pulse is 98/min. Her oropharynx is clear and there is no thrush or
12 lymphadenopathy. Her abdomen is benign and the pelvic examination is w ithin normal limits. She has pain
13 along the tendon sheaths w ith active and passive hand movement. A photo of her skin rash is show n below .
14
15
16
17
18
19
20
21
22
23
24
25
26
?.7 Dlployod wltll perm/nlon from Current Medklne Group LLC@ 2000.
18
19 W hich of the follow ing is the most likely cause of her symptoms?
20
21 A. Lyme disease [12%]
22
23 B. Syphilis [8%]
24 ~ C. Gonococcemia [68%]
25 D. Meningococcemia [3%]
26
?.7 E. Acute HIV infection [4%]
18 F. Acute rheumatic fever [4%]
19
20
21 Explanation: User ld:
22
23 This patient has the classic triad of polyarthralgia, tenosynovitis, and painless vesiculopustular skin lesions for
24 disseminated gonococcal infection. Her complaints of w rist, elbow and ankle pain are consistent w ith
25 gonococcal polyarthralgias, and her pain elicited along the tendon sheaths suggests tenosynovitis. The
26 lesions show n in the photograph are typical vesiculopustular lesions of disseminated gonococcal infection;
?.7 they tend to number from two to ten and may be dismissed as furuncles or pimples. Fevers and chills may
18 be present. A history of recent unprotected sex w ith a new partner is frequently associated w ith disseminated
19 gonococcal infection, w hile symptoms of symptomatic venereal disease are most often absent.
20
(Choice A) Arthritis and rash are associated w ith Lyme disease. The rash (erythema migrans} of Lyme
21
disease occurs early on in illness, w hile arthritis, typically monoarticular knee pain, does not occur until
22
months later. Erythema migrans occurs at the site of a tick bite, and has a "bulls eye" appearance.
23
Tenosynovitis is not typical of Lyme disease.
18
19 (Choice B) The skin findings of syphilis include a painless ulcer of the genitalia that occurs in the primary
20 stage of infection, and a maculopapular rash of the palms and soles in the secondary stage.
21
22 (Choice D) Meningococcemia presents w ith a petechial rash, high fever, headache, nausea/vomiting, stiff
23 neck, and photophobia . This patient specifically denies many of these symptoms. Furthermore, arthritis,
24 tenosynovitis, and the rash in the image are not consistent w ith meningococcemia.
25
26 (Choice E) The symptoms of acute HIV infection include fever, arthralgias, sore throat, lymphadenopathy,
27 mucocutaneous lesions, diarrhea, and w eight loss. Her symptoms are more consistent w ith disseminated
28 gonococcal infection. All patients w ith disseminated gonococcal infection, how ever, should undergo HIV
29 screening.
30
(Choice F) An episode of pharyngitis typically precedes the onset of acute rheumatic fever by 2-4 w eeks.
31
Symptoms of acute rheumatic fever include migratory arthritis of the large joints, erythema marginatum
32
(raised ring-shaped lesions over the trunk and extremities}, subcutaneous nodules, carditis, and Sydenham
33
34 chorea .
35 Educational objective:
36 Disseminated gonococcal infection often presents w ith a triad of polyarthralgias, tenosynovitis, and
37 vesiculopustular skin lesions.
38
39
Time Spent: 19 seconds Copyright UW orld Last updated: [2/19/2015]
40
1
2
" Item: 12 of 44
Q. ld: 3575 [
IIPMark <J
Previous
[:>
Next
jf
Lab Values
~
Notes
~
Calculator
3
4
5
6 A 45-year-old tennis player comes to your office w ith a complaint of pain over the lateral side of the right
7 elbow. He has been a professional tennis player for 15 years but has never had this kind of pain before.
8 Range of motion at both elbow s is normal. There is point tenderness over the lateral side of the distal end of
9 right humerus. Pain is exacerbated by extension of w rist against resistance. The rest of the physical
10 examination is normal. W hich of the follow ing is the most likely diagnosis in this patient?
11

13 .; 0 A. Lateral epicondylitis [89%]


14 B. Rotator cuff injury [2%]
15 C. Radial tunnel syndrome [4%]
16
17 D. Posterior interosseous nerve entrapment [3%]
18 E. Rupture of long head of biceps tendon [2%]
19
20
21 Explanation: User ld:
22
The patient described is suffering from lateral epicondylitis (tennis elbow ). Though the condition is termed
23
"tennis elbow", lateral epicondylitis can result from a variety of activities that involve repeated forceful w rist
24
extension and supination (backhand in tennis, use of a screw driver). Patients w ith lateral epicondylitis present
18 w ith pain near the lateral epicondyle that is w orsened by use. Examination reveals point tenderness just distal
19 to the lateral epicondyle and production of pain during active extension of the w rist. The underlying
20 pathophysiology is degeneration of the extensor carpi radialis brevis tendon near the lateral epicondyle.
21
22 (Choice B) Rotator cuff injury results in shoulder pain, w eakness and decreased range of motion. It is
23 typically the result of impingement of the supraspinatus tendon.
24
25 (Choice C) Radial tunnel syndrome causes signs and symptoms similar to lateral epicondylitis and may
26 occur in conjunction w ith that condition. The tenderness in radial tunnel syndrome tends to overlie the
27 extensor muscle w ad, and pain in this syndrome is elicited on examination by flexing the patient's long finger
28 w hile the patient actively extends the fingers and w rists.
29
30 (Choice D) Posterior interosseous nerve entrapment results in w eakness of the extrinsic extensors of the
31 hand and fingers.
32
(Choice E) Rupture of long head of biceps tendon causes pain in the upper arm and shoulder and a
33
prominent bulge in the midportion of the upper arm .
34
35 Educational objective:
36 Lateral epicondylitis (tennis elbow ) manifests as pain w ith supination or extension of the w rist and point
37 tenderness just distal to the lateral epicondyle.
38
39
Time Spent: 3 seconds Copyright UW orld Last updated: [2119/2015]
40
1 ...
2 Item: 13 of 44
Q. ld: 3164 [
III P Mark -<J
Previous
t:>
Ne><t
jf
Lab Values
~
Notes
~
Calculator
3
4
5
6 A 68-year-old w oman comes to the emergency department in acute distress. She complains of the sudden
7 onset of complete vision loss in her right eye. Her vision had been blurry for the past few days and acutely
8 w orsened an hour ago. The patient also describes 2 months of right-sided headaches for which she took
9 ibuprofen without much relief. She has no nausea or vomiting but does have malaise and fatigue. Her
10 medical problems include diet-controlled type 2 diabetes mellitus, hypertension, and degenerative joint
11 disease. Her temperature is 37.2 C (98.9 F), blood pressure is 146/86 mm Hg, and heart rate is 78/min.
12 Examination show s complete loss of vision in the right eye. Pupils are 4 mm bilaterally. Funduscopy show s
a sw ollen pale disc with blurred margins. The heartbeat is regular, and a bruit is heard in the right
14 subclavicular area. Motor and sensory examination is within normal limits. Erythrocyte sedimentation rate is
15 85 mm/h. Which of the following is the most appropriate next step in management of this patient?
16
17
.- A. High-dose acetazolamide [5%1
18
19 ., B. High-dose intravenous methylprednisolone [79%1
20 C. Low -dose oral prednisone [3%1
21
D. Methotrexate therapy [0%]
22
23 E. MRI of the brain [3% 1
24 F. Temporal artery biopsy [10%1
25
26
?.7 Explanation: User ld:
18
19
I
20 Giant cell arteritis - clinical manifestations
21
22 Systemic
23 symptoms Fever, fatigue, malaise, weight loss
24
25
26 Headaches: Located in temporal areas
?.7 Jaw claudication: Most specific symptom of GCA
18
19
20
21
22
23
24
Localized
symptoms
PMR
Arm claudication: Associated bruits in subclavian
or axillary areas
Aortic wall thickening or aneurysms
I
CNS: TIAs/stroke, vertigo, hearing loss
25
26
?.7 Amaurosis fugax: Transient vision field defect
18 Visual progressing to monocular blindness
19 symptoms
20 AION: Most common ocular manifestation
21
22 Nonmochromic anemia
23 Laboratory
24 Elevated ESR & CRP
results
25 Temporal artery biopsy
26
?.7
18 PMR only: Low-dose oral glucocorticoids
19 (eg, prednisone 10-20 mg daily)
20
21 GCA: Intermediate- to high-dose oral glucocorticoids
22 (eg, prednisone 40-60 mg daily)
Treatment
23 GCA with vision loss: Pulse high-dose IV
24 glucocorticoids (eg, methylprednisolone 1000 mg daily)
25 for 3 days followed by intermediate- to high-dose
26
oral glucocorticoids
?.7
18
AION = anterior ischemic optic neuropathy; CNS =central nervous system;
19 CRP = C-reactive protein; ESR= erythrocyte sedimentation rate; GCA =giant cell arteritis;
20 PMR = polymyalgia rheumatica; TIA = transient ischemic attack.
21
~UWortd
22
23 This patient's clinical presentation, with headaches, sudden loss of vision, abnormal funduscopic findings, and
24
elevated erythrocyte sedimentation rate, is suggestive of giant cell arteritis (GCA). GCA (also referred to as
25 temporal arteritis) is a chronic vasculitis affecting the medium and large vessels and is characterized by a
26 variety of localized, systemic, and ocular manifestations. Physical examination may show diminished pulses
?.7 and/or bruits on auscultation of the axillary, brachial, or femoral arteries and over the supraclavicular and/or
18 carotid areas. Anterior ischemic optic neuropathy is the most common ocular manifestation and is
19 detected on funduscopy by the presence of a sw ollen and pale disc with blurred margins.
20
21 Visual syndromes with vision loss are the most dreaded complication of GCA, and blindness can develop in
22 up to 25%-50% of untreated patients. In patients with suspected GCA and visual loss, the most appropriate
23 next step is to initiate therapy with high-dose intravenous corticosteroids, follow ed by oral therapy with a

I
24 slow taper over several months. Immediate therapy with systemic corticosteroids reduces the progression of
25 visual complications in the affected and unaffected eyes; it should be started promptly while awaiting
26 confirmation of the diagnosis.
?.7
18 (Choice A) Acetazolamide is used to treat open-angle glaucoma and benign intracranial hypertension. ~ has
19 no role in the management of patients with suspected GCA.
20
21 (Choice C) Initial therapy with low -dose prednisone (10-20 mg daily) is recommended for patients with
22 polymyalgia rheumatica without evidence of GCA. Higher-dose oral prednisone (40-60 mg/day) is
23 recommended only in patients with uncomplicated GCA without associated symptoms or signs of end-organ
24 damage.
25
00

18
19
(Choice D) Methotrexate is less effective than high-dose glucocorticoids in treating temporal arteritis. It is
used primarily as a steroid-sparing agent for patients who develop significant adverse effects with systemic
glucocorticoids.
-
20
21 (Choice E) MRI with contrast enhancement can identify vessel w all edema and provide a noninvasive
22 method of diagnosis in patients with suspected GCA. How ever, immediate empiric treatment takes
23 precedence in this patient, and temporal artery biopsy remains the definitive method for diagnosis.
24
25 (Choice F) Temporal artery biopsy is indicated to confirm the diagnosis of GCA. How ever, therapy with
26 systemic glucocorticoids should be started immediately when the diagnosis is suspected and should not be
27 withheld while awaiting confirmation with temporal artery biopsy.
28
Educational objective:
29
Giant cell arteritis (temporal arteritis) should be suspected in patients with temporal headaches, jaw
30
claudication, and vision loss. Patients with suspected giant cell arteritis who have visual symptoms should be
31
started immediately on high-dose systemic glucocorticoids to reduce the progression of visual complications.
32
33
34 References:
35 1. Visual improvement with corticosteroid therapy in giant cell arteritis. Report of a large
36 study and review of literature.
37
38
39
Time Spent: 2 seconds Copyright UW orld Last updated: [11 / 10/20141
40
. -.--- - ..-- - - -~- -
1
2
" Item: 14 of 44
Q. ld: 3515 [
IIPMark <J
Previous
[:>
Next
jf
Lab Values
~
Notes
~
Calculator
3
4
5
6 A 22-year-old Caucasian female comes to your office complaining of difficulty swallow ing. She says that solid
7 food sticks in the middle of her chest, and that's w hy she prefers liquids. She has lost 10 pounds over the last
8 3 months. She also complains of recent severe heartburn that does not respond w ell to over-the-counter
9 antacids. On review of systems, she denies cough, shortness of breath and palpitations. She has noticed
10 occasional sw elling and pain in her small finger joints. Her fingers turn blue upon cold exposure, and she
11 always w ears gloves to keep them w arm . She does not smoke or drink alcohol. She denies illegal drug use.
12 W hich of the follow ing is the most likely diagnosis?
13

15 .; A. CREST syndrome [95%]


16 B. Esophageal neoplasm [1 %]
17 C. Achalasia [1 %]
18
19 D. Diffuse esophageal spasm [ 1%]
20 E . Rheumatoid arthritis [1 %]
21
22
23 Explanation: User ld:
24
CREST syndrome refers to a constellation of findings including Calcinosis cutis, Raynaud phenomenon,
25
Esophageal dysmotility, Sclerodactyly and Telangiectasias. The patient described has at least three of these
26
findings. CREST syndrome is a limited form of systemic sclerosis. Calcinosis cutis refers to localized
?.7
dystrophic deposition of calcium in the skin and manifests as subcutaneous pink-to-w hite nodules typically on
18
the upper extremities. These may ulcerate and drain a chalky material. Raynaud phenomenon refers to
19
acute bouts of digital ischemia in response to cold and may lead to digital necrosis. The fingers typically
20
progress through w hite, blue and hyperemic stages. Esophageal dysmotility classically results in complaints
18 of food being "stuck" in the esophagus and GERD. Sclerodactyly describes fibrosis of the skin of the fingers
19 distal to the metacarpophalangeal joints resulting in a shiny appearance of the skin and flexion contractures
20 and joint pain. Finally, telangiectasias in CREST manifest as "mat-like" patches most commonly on the face
21 and palms.
22
23 (Choice B) An esophageal neoplasm or diverticulum may cause difficulty sw allow ing but w ould not explain
24 the other symptoms.
25
26 (Choice C) Achalasia causes dysphagia for both solids and liquids. A barium esophagogram w ould show the
27 "bird's beak" sign in the distal esophagus.
28
29 (Choice D) Diffuse esophageal spasm may cause acute and severe retrosternal chest pain as w ell as
30 dysphagia due to uncoordinated and I or hypertensive esophageal contractions.
31
(Choice E) Rheumatoid arthritis is associated w ith bilateral, symmetric joint pain that classically first affects
32
the proximal interphalangeal joints. Patients may also have rheumatoid nodules, pulmonary fibrosis and
33
infiltrate, scleritis and, rarely, splenomegaly w ith neutropenia (Felty syndrome).
34
35 Educational objective:
36 CREST syndrome refers to a constellation of findings including Calcinosis cutis, Raynaud phenomenon,
37 Esophageal dysmotility, Sclerodactyly and Telangiectasias.
38
39
Time Spent: 3 seconds Copyright UW orld Last updated: [12129/2014]
40
1
2
" Item: 15 of 44
Q. ld: 3176 [
IIPMark <J
Previous
[:>
Next
jf
Lab Values
~
Notes
~
Calculator
3
4
5
6 A 35-year-old African-American w oman comes to the physician's office complaining of blurred vision, cough
7 and shortness of breath. For the past few days she has had mild fevers, malaise and easy fatigability. She
8 has never had these symptoms before and is anxious to uncover a diagnosis. She w as recently incarcerated
9 for two months. She practices unprotected sex with her new boyfriend. Her temperature is 37.2 C (98.9 F)
10 and her blood pressure is 116/80 mmHg. On exam, her right eye is red and slit lamp examination show s
11 leukocytes in the anterior chamber. Lungs have patchy rales. Chest x-ray show s bilateral reticulonodular
12 infiltrates and hilar adenopathy. Which of the following is the most likely diagnosis in this patient?
13
14
A. Acute HIV infection [1 0%]
16 B. Disseminated tuberculosis [9%]
17 ~ C. Sarcoidosis [76%]
18
D. Histoplasmosis [2%]
19
20 E. Ankylosing spondylitis [1 %]
21 F. Hodgkin's lymphoma [1 %]
22
23
24 Explanation: User ld:
25
26 This patient's presentation is best explained by sarcoidosis. Sarcoidosis is a systemic illness that most
?.7 commonly afflicts African-American w oman in the third and fourth decades of life. Half of cases are
18 diagnosed based on incidental findings on chest x-rays. In those patients who present symptomatically, the
19 most common complaints are cough, dyspnea, fever, and w eight loss. While the lungs are the most
20 frequently involved organ system, the skin (erythema nodosum), eyes (anterior uveitis), joints (arthritis), and
21 many other organ systems may be affected. Classic findings on chest x-ray include bilateral hilar adenopathy
22 and reticular opacities, both of which are present in this patient. The description of a red eye with leukocytes
23 in the anterior chamber is consistent with anterior uveitis.
24
(Choice A) This patient is at some risk of acute HIV infection given her unprotected sex with a new partner.
25
Symptoms of acute HIV infection are non-specific and include fever, arthralgias, sore throat,
26
lymphadenopathy, mucocutaneous lesions, diarrhea, and w eight loss. The findings on x-ray and
?.7
ophthalmologic exam are inconsistent with acute HIV infection.
18
19 (Choice B) Recent incarceration is a risk factor for tuberculosis (TB). On chest x-ray, primary TB presents
20 with low er lobe infiltrates while reactivation presents with apical nodules or cavitary lesions. Cough,
18 hemoptysis, night sweats, w eight loss, and fever are common. Disseminated disease may affect any organ
19 system. This patient's chest x-ray, signs and symptoms are more suggestive of sarcoidosis.
20
21 (Choice D) Histoplasmosis is most common in the Ohio River Valley and primarily affects the lungs.
22 Histoplasmosis may present similarly to both sarcoidosis and TB in terms of pulmonary symptoms and
23 radiographic findings. How ever, anterior uveitis is not consistent with histoplasmosis. Rather, damage to the
24 retina is the most common ocular manifestation in histoplasmosis.
25
26 (Choice E) Anterior uveitis frequently occurs in ankylosing spondylitis (AS). How ever, AS presents with pain,
27 stiffness and limited range of motion of the low er back. Pulmonary infiltrates and hilar adenopathy are not
28 characteristic of AS.
29
(Choice F) Hodgkin's lymphoma is most common in young adults. It typically presents with painless, sw ollen
30
lymph nodes, and hilar adenopathy. "B symptoms" of fever and w eight loss may be present. The absence of
31
peripheral lymphadenopathy and the occurrence of anterior uveitis make sarcoidosis more likely in this patient.
32
33 Educational objective:
34 Sarcoidosis is most common in African-American w oman during the third and fourth decades of life. The
35 lungs are the most commonly affected organ system. When symptomatic, cough, erythema nodosum,
36 anterior uveitis, and arthritis may be seen . Hilar adenopathy and reticular opacities on chest x-ray are classic
37 findings.
38
39
Time Spent: 3 seconds Copyright UW orld Last updated: [2119/2015]
40
1
2
" Item: 16 of 44
Q.ld: 4572 [
l ll''Mark <:J
Previous
t>
Next
il
Lab Values
~
Notes
~
Calculator
3
4
5
6 A 43-year-old w oman comes to the physician complaining of bilateral joint pain and sw elling in her hands for
7 the past few months. She reports easy fatigability and loss of energy that has w orsened insidiously. It is
8 especially difficult for her to perform daily activities in the morning due to prolonged stiffness. The patient also
9 describes frequent knee pain accompanied by a low -grade fever. She takes over-the-counter ibuprofen to
10 relieve her symptoms. Her hematocrit is 33% and creatinine is normal. This patient is at greatest risk for
11 which of the following?
12
13
A. Avascular bone necrosis [13%]
14
15 B. Osteitis deformans [42%]
C. Osteitis fibrosa cystica [11 %]
17
., D. Osteoporosis [31%]
18
19 E. Osteosarcoma [3%]
20
21
Explanation: User ld:
22
23
24
25 Clinical feat ures of rheumatoid arthritis
26
?.7 Symptoms
18
19
20
21
Insidious onset, multiple joint pain, stiffness & swelling
Morning stiffness lasting hours, improves with activity
I
22 Small joints (eg, PIP, MCP, MTP) commonly involved
23 Monoarthritis (eg, knees, elbows) can also occur later
24
25 Spares the DIP j oint, unlike osteoarthritis
Clinical
26 presentation Examination
?,7
Affected joints are tender to the touch, swollen, with limited range
18
19
of motion
20 Tenosynovitis of the palms ? "trigger finger"
21
Rheumatoid nodules (especially on elbows)
22
23 Cervical joint involvement can lead to spine subluxation ? spinal
24 cord compression
25
26
?.7 Positive anti-CCP antibodies (diagnostic testing)
Laboratory/
18 High lgM rheumatoid factor
imaging
19
studies High C-reactive protein & ESR correlate with disease activity
20 X-ray: Soft-tissue swelling, joint space narrowing & bony erosions
21
22 Anti-CCP = anti-cyclic citrullinated peptide: DIP = distal interphalangeal joints:
23 ESR =erythrocyte sedimentation rate; lgM = Immunoglobulin M; MCP =metacarpophalangeal;
24 MTP = metatarsophalangeal.
25 @UWorld
26
?,7 This patient has a characteristic presentation of rheumatoid arthritis (RA). RA affects women more often
18 than men and has a typical onset at age 30-50. Morning stiffness for ~1 hour is typical of RA and w ould
19 explain this patient's difficulty with morning activities. Symmetric joint swelling is characteristic. The most
20 commonly affected joints include the metacarpal-phalangeal joints, proximal interphalangeal joints, w rists,
21 and knees. Due to the autoimmune nature of the disease, systemic symptoms such as fatigue, fever, and
22 anemia often occur. Patients are started on treatment (including disease-modifying agents) based on the
23 severity of their symptoms.
24
25 Increased levels of proinflammatory cytokines, corticosteroid therapy, and lack of physical activity may
26 contribute to local (around inflamed joints) or generalized loss of bone mass in RA patients. There is an
?.7 increased risk of osteopenia , osteoporosis, and bone fractures, especially if other risk factors (eg, low body
18 weight, female sex, family history of osteoporosis, cigarette smoking, postmenopausal state, excessive
19 alcohol use, other comorbidities) are present. The degree of bone loss generally correlates with disease
20 activity. Strategies to prevent bone loss (adequate physical activity, optimization of vitamin D and calcium
21 intake, minimization of glucocorticoid dose) should be implemented. Most experts suggest a low threshold for
22 starting bisphosphonate therapy in RA patients.
23
24 Although there may be associations betw een RA and some of the other answ er options, the risk of
25 osteoporosis is the greatest (up to a 2-fold increase, with one study showing a prevalence of 22%).
26
(Choice A) Avascular necrosis of bone is most commonly seen in patients with systemic corticosteroid
?,7
therapy, heavy alcohol use, systemic lupus erythematosus, or sickle cell disease. Corticosteroids are often
18
used in patients with RA. Avascular necrosis is therefore possible in this patient, but osteoporosis would be
19
more likely.
20
21 (Choice B) Paget disease of bone (osteitis deformans) is a condition of increased bone turnover due to
22 osteoclast overactivity, leading to the replacement of lamellar bone with abnormal w oven bone. The risk of
23 Paget disease is not increased in RA.


24
25 (Choice C) Osteitis fibrosa cystic a (Von Recklinghausen disease of bone), which presents with bony pain, is
18 characterized by excessive osteoclastic resorption of bone, leading to replacement with fibrous tissue (brown
19 tumors). It is very rare and is seen primarily in patients with parathyroid carcinoma. It can also occur in
20 primary and secondary/tertiary (advanced renal disease) hyperparathyroidism. Renal injury may occur in RA
21 (from use of nonsteroidal anti-inflammatory drugs and some disease-modifying agents or from amyloidosis
22 due to inflammation), but the risk of osteoporosis is far greater than the risk of osteitis fibrosa cystic a.
23
24 (Choice E) Osteosarcoma is a primary malignant bone tumor. Risk factors include Paget disease and
25 radiation and chemotherapy exposure.
26
Educational objective:
27
Patients with rheumatoid arthritis are at increased risk of developing osteopenia, osteoporosis, and bone
28
fractures, especially if additional risk factors for osteoporosis are present. Management includes adequate
29
physical activity, optimization of calcium and vitamin D intake, minimization of corticosteroid therapy, and
30
consideration for bisphosphonate treatment.
31
32
33 References:
34 1. Clinical decision rules in rheumatoid arthritis: do they identify patients at high risk for
35 osteoporosis? Testing clinical criteria in a population based cohort of patients with
36 rheumatoid arthritis recruited from the Oslo Rheumatoid Arthritis Register.
37
38
39
Time Spent: 4 seconds Copyright UWorld Last updated: [12/ 14/2014]
40
1
2
" Item: 17 of 44
Q. ld: 3311 [
IIPMark <J
Previous
[:>
Next
jf
Lab Values
~
Notes
~
Calculator
3
4
5
6 A 44-year-old w hite male presents w ith a long history of joint pains in several joints. He has seen a physician
7 before but no diagnosis w as made. He has been taking ibuprofen w ith partial relief. He has now developed
8 fever, diarrhea and w eight loss. He denies any genitourinary or eye symptoms. He does not use tobacco,
9 alcohol or drugs. He is a farmer. On examination, he has generalized lymphadenopathy and non-deforming
10 arthritis. Small intestinal biopsy reveals periodic acid-Schiff (PAS}-positive macrophages. W hich of the
11 follow ing is the most likely diagnosis?
12
13
14 A. Reactive arthritis [4%1
15 B. Sarcoidosis [3%1
16 C. Inflammatory bow el disease [6%1

18 v D. W hipple's disease [78%1


19 E. Celiac disease [5%1
20 F. Intestinal lymphoma [3%1
21
22 G. HIV Infection [ 1%1
23
24 Explanation: User ld:
25
26 The patient described is most likely suffering from W hipple disease. W hipple's disease is a multisystem
disease that is characterized by a multitude of possible manifestations. Most commonly, patients w ith
18 W hipple disease present w ith a history of chronic malabsorptive diarrhea (steatorrhea, flatulence, abdominal
19 distention), protein-losing enteropathy, w eight loss, migratory non-deforming arthritis, lymphadenopathy and a
20 low -grade fever. The disease may also cause damage to the eye, CNS and m yocardium . W hipple's disease
21 is caused by infection w ith the gram-positive bacillus Tropheryma v.hippelii, but a mechanism w hereby
22 patients contract this infection has yet to be determined. The diagnosis is made w ith a small intestinal biopsy
23 and PCR in patients w ith clinical symptoms consistent w ith the disease. Small intestinal biopsy show s PAS-
24 positive macrophages in the lamina propria containing non-acid-fast gram-positive bacilli.
25
26 (Choice A) Reactive arthritis classically occurs follow ing an infection. The classic example is - HLA-827-
associated reactive arthropathy characterized by conjunctivitis, urethritis and arthritis. It typically follow s
18 infections by Chlamydia, Shigella, Salmonella, Yersinia, Campylobacter and C. difficile.
19
(Choice B) In sarcoidosis biopsy show s non-caseating granulomas.
18
19 (Choice C) On histopathology, ulcerative colitis show s acute and chronic inflammation of the mucosa leading
20 to crypt abscess formation. Crohn disease show s full-thickness inflammation w ith granuloma and lymphoid
21 aggregate formation and skip regions.
22
23 (Choice E) Celiac disease (celiac sprue, gluten-sensitive enteropathy) is an autoimmune disease
24 characterized by malabsorptive diarrhea. Serologic testing show s anti-endomysia! and anti-transglutaminase
25 antibodies, and histopathology show s effacement of small intestinal villi.
26
27 (Choice F) Intestinal lymphoma usually presents w ith abdominal pain, w eight loss, nausea and vomiting,
28 distention, and occult blood in the stool. Malabsorption is not a typical feature, and histopathology w ould show
29 a diffuse infiltrate by atypical lymphocytes.
30
(Choice G) HIV Infection may be associated w ith a similar syndrome of fever, malabsorptive diarrhea and
31
w eight loss w hen Mycobacterium aviurrrintracellulare infects the small intestinal w all. This w ould also cause
32
PAS-positive macrophages on small intestinal biopsy, but these bacilli w ould also be acid-fast.
33
34 Educational objective:
35 W hipple disease is a multisystem disorder w ith a varied presentation caused by infection w ith the gram-
36 positive bacillus Tropheryma v.hippelii Chronic malabsorptive diarrhea, w eight loss, migratory non-deforming
37 arthritis, lymphadenopathy and a low -grade fever are the most common presenting symptoms.
38
39
Time Spent: 3 seconds Copyright UW orld Last updated: [2119/20151
40 "
1 ...
2 Item: 18 of 44
Q.ld: 3153 [
III P Mark -<J
Previous
t:>
Ne><t
jf
Lab Values
~
Notes
~
Calculator
3
4
5
6 A 28-year-old w oman comes to the physician with pain in her left knee joint. She had mild discomfort and pain
7 in her right wrist 4 days ago and left ankle pain 2 days ago. She has no recent respiratory illness, diarrhea, or
8 urinary symptoms. She has no vaginal discharge. The patient has no previous medical problems and takes
9 no medications. She drinks half a pint of vodka daily but does not use intravenous drugs. She is single and
10 sexually active. Her last menstrual period w as a w eek ago. Temperature is 38.5 C (101 .3 F), blood pressure
11 is 120/80 mm Hg, pulse is 98/min, and respirations are 15/min. Examination of the knee show s w armth,
12 tenderness, decreased range of motion, and an effusion. No skin lesions are present and her pelvic
13 examination is unremarkable. Synovial fluid analysis show s a white blood cell count of 50,000/mm'. Which of
14 the following is the most likely cause of her symptoms?
15
16
~ A. Acute HIV infection [1 %]
17
B. Acute rheumatic fever [2%]
19 C. Crystal-induced arthritis [6%]
20
21 >1 D. Gonococcal septic arthritis [65%]
22 E. Infective endocarditis [0%]
23 F. Non-gonococcal septic arthritis [19%]
24
25 G. Reactive arthritis [7%]
26
?.7 Explanation: User ld:

19 I
20
Disseminated gonococcal infection
21
22
23 Purulent arthritis without skin lesions
24
25 OR
26 Clinical Triad of:
?.7 presentation Tenosynovitis (eg, wrist, ankles, fingers & knees)
19 Dermatitis (pustules, maculas, papules & bullae)
20
Migratory asymmetric polyarthralgia without purulent arthritis
21
22
23 Blood cultures (2 sets) but may be negative
24
25 Synovial fluid analysis may show up to 50,000 cells/mm 3
26 Diagnosis Urethral, cervical, pharyngeal or rectal cultures
?.7
Recommend HIV & syphilis screen
19 Recurrent DGI: check terminal complement activity
20
21
22 IV ceftriaxone 1 g/day for 7-14 days, switch to PO (cefixime)
23 when clinically improved
24
Joint drainage for purulent arthritis
25 Treatment
26 Empiric azithromycin (single 1-g dose) OR doxycycline for
?.7 7 days for concomitant chlamydia! infection
19 Treat sexual partners
20
21 OGI = disseminated gonococcal infection; HIV = human immunodeficiency virus; IV = intravenous; PO = orally.
22 @UWorld
23
24 The fever, knee pain, and white blood cell (WBC) count of 50,000/mm' on synovial fluid analysis strongly
25 suggest septic arthritis. Septic arthritis in a young, sexually active individual is most often caused by
26 N eisseria gonorrhoeae (75% of cases).
?.7
Gonococcal septic arthritis may present with asymmetric polyarthralgias (most often associated with
19 tenosynovitis and skin rash) or an isolated purulent mono- or oligoarthritis. In some patients, asymmetric
20 polyarthralgias may precede purulent monoarthritis. Roughly 75% of cases are "silent," meaning that the
21 preceding genitourinary or pharyngeal infection goes unnoticed.
22
23 For this patient, migratory asymmetric polyarthralgias of the right wrist and left ankle preceded purulent
24 monoarthritis of the left knee. Although skin rash and tenosynovitis are not present in this patient, purulent
25 arthritis in a sexually active individual is gonococcal arthritis until proven otherwise. Synovial fluid
26 white blood cell count is about 50,000/mm' (slightly low er than other septic arthritides). Gram stain of the
?.7 synovial fluid (positive in 25% of cases), blood cultures (positive in 20%-50% of cases), and genital/pharyngeal
mucosal nucleic acid amplification tests (positive in 90% of cases) are used to confirm the diagnosis.
19 Treatment is ceftriaxone or cefotaxime.
20
21 (Choice A) Symptoms of acute HIV infection tend to present 2-4 w eeks after exposure to the virus. The
22 symptoms are nonspecific and include fever, arthralgias, sore throat, lymphadenopathy, mucocutaneous
23 lesions, diarrhea, and w eight loss.

I
24
(Choice B) Acute rheumatic fever generally occurs 2-4 w eeks after pharyngitis caused by Group A
25
Streptococcus. Symptoms of acute rheumatic fever include migratory arthritis of the large joints, erythema
26
marginatum rash, subcutaneous nodules, carditis, and Sydenham chorea .
?.7
(Choice C) Crystal-induced arthritis often presents with a w arm, painful, swollen joint accompanied by
19 low -grade fever. Synovial fluid analysis show s a WBC count of 5,000-80,000/mm' and characteristic crystals
20 under polarized light.
21
~4 (Choice E) Arthritis is not a prominent symptom of infective endocarditis. Fever, a new heart murmur, Osier
nodes (nodules on finger and toe pads), Roth spots (retinal hemorrhages), and Janew ay lesions (small
19 hemorrhages on extremities) are classic.
20
21 (Choice F) Non-gonococcal septic arthritis is less likely than gonococcal arthritis in a young, sexually active
22 patient. A more classic profile for non-gonococcal arthritis w ould be an adu~ with a chronic disease (eg,
23 diabetes, osteoarthritis) who generally has single-joint involvement (rather than migratory polyarthralgias) and
24 fever. Gram stain and blood cultures are necessary for definitive diagnosis.
25
26 (Choice G) Reactive arthritis may be caused by genitourinary infection with Chlamydia trachoma/is or by
27 certain gastrointestinal infections. It presents with a triad of arthritis, conjunctivitis, and urethritis.
28
Educational objective:
29
N eisseria gonorrhoeae is the most common cause of septic arthritis in young, sexually active patients.
30
Gonococcal septic arthritis may present with asymmetric polyarthralgias (often associated with tenosynovitis
31
and skin rash) or as an isolated purulent mono- or polyarthritis. Diagnosis may be confirmed by Gram stain of
32
the synovial fluid, blood cultures, and genital/pharyngeal mucosal nucleic acid amplification tests.
33
34
35 References:
36 1. Gonococcal and nongonococcal arthritis.
37
38
39
Time Spent: 2 seconds Copyright UW orld Last updated: [1 0/30/2014]
40
Media Exhibit

Joint fluid analysis I


Joint flu id Non-inflammatory Inflammatory
Normal Septic joint
characteristics (e.g., OA) (e.g., crystals, RA)

Translucent or
Fl uid clarity Clear Clear Opaque
opaque

50,000-
WBCs/mm3 < 200 200-2,000 2,000-1 00,000
150,000

Often >
PMNs < 25% 25% Often> 50%
80-90%
USMlEWorld.. LLC

~
Media Exhibit

Acute rheumatic fever I


"
Acute rheumatic fever

Peak incidence: Age 5-15


Epidemiology
Twice as common in girls

Joints (migratory arthritis)


(Carditis)
Major Nodules (subcutaneous)
Erythema marginatum
Sydenham chorea
Clinical
features
Fever
Arthralgias
Minor Elevated erythrocyte
sedimentation rate/
C-reactive protein
Prolonged PR interval

Late Mitral regurgitation/stenosis


sequelae

Penicillin for group A streptococcal


Prevention
(Streptococcus pyogenes) pharyngitis
@UWo ~d

"
~
1
2
" Item: 19 of 44
Q.ld: 4580 [
l ll''Mark <:J
Previous
t>
Next
il
Lab Values
~
Notes
~
Calculator
3
4
5
6 A 51-year-old w oman comes to the clinic w ith 6 months of fatigue and low back pain radiating to the buttocks.
7 She also complains of persistent muscle pain in her arms and shoulders that w orsens acutely after exercise.
8 Physical examination show s normal muscle strength. Her joints are not sw ollen, but palpation over the outer
9 upper quadrants of the buttocks and the medial aspect of the knees elicits tenderness. Her erythrocyte
10 sedimentation rate is 12 mm/hr. W hich of the following is the most likely diagnosis?
11
12
13 >1 A. Fibromyalgia [61%1
14 B. Giant-cell arteritis [0%1
15 C. Polyarteritis nodosa [1 %1
16
17 D. Polym yalgia rheumatic a [21 %1
18 E. Polym yositis [1 0%1
F. Rheumatoid arthritis [0%1
20
21 G. Seronegative spondyloarthropathy [6%1
22
23 Explanation: User ld:
24
25
26 Condition Distinguishing clinical features Laboratory/diagnosis
?.7
18

20
21
. Usually young-to-midd le-aged women . No abnormal laboratory studies
. Widespread musculoskeletal pain in . Possible tender points on physical I
both sides of body, above & below waist examination (eg, mid trapezius,
22
23
. Fatigue when arising from sleep & mid- lateral epicondyle, costochondral
24 Fibromyalgia afternoon junction)
25 . Cognitive difficulties (eg, attention/tasks . Symptoms >3 months with
26
requiring rapid thought changes) increased widespread pain index
?,7
18
. Nonspecific gastrointestinal symptoms or symptom severity score
(eg, diarrhea, constipation)
20
21
22
. Symmetrical proximal muscle weakness . Elevated muscle enzymes
23 . Increasing d ifficulty climbing stairs, (eg, creatine kinase, aldolase,
24 getting up from a chair, carrying heavy lactate dehydrogenase, aspartate
25 Polymyositis
groceries aminotransferase)
26
. Less prominent hip/shoulder . Possible autoantibodies
?.7
18 involvement . Electromyography abnormal
20 . Age usually >50 . Clinical diagnosis
21
22
. Aching & morning stiffness >pain in . Significantly elevated eryth rocyte
23 shoulders, hips, neck, torso sedimentation rate
24
Polymyalgia
. Synovitis, bursitis, decreased range of . Symptoms improve with
25 rheumatica
motion corticosteroids
26
?,7
. No significant muscle tenderness
18 . Possible systemic symptoms
) USMLEV/01Id, llC
20
21
This patient's presentation is consistent w ith likely fibromyalgia (FM). FM presents most commonly in
22
young-to-middle-aged w omen w ith w idespread pain, fatigue, and cognitive/mood disturbances. Patients tend
23
to have a fairly normal physical examination except for point muscle tenderness in areas such as mid
24
trapezius, lateral epicondyle, costochondral junction in the chest, and greater trochanter. Patients perceive
25
that their pain and fatigue w orsen acutely after exercise. How ever, studies show that gradual and incremental
26
low -impact exercises (eg, fast w alking, sw imming, w ater aerobics) can improve pain and fatigue and provide
?.7
long-term benefit.
18
FM has no specific diagnostic laboratory findings and the diagnosis is primarily based on history and exam
20 findings. Symptoms frequently overlap w ith those of other disorders (eg, chronic fatigue syndrome, irritable
21 bow el syndrome). Revised 2010 American College of Rheumatology criteria suggest using the widespread
22 pain index and symptom severity score rather than trigger points for FM diagnosis. The index and score
23 better emphasize cognitive problems, fatigue, and severity of somatic symptoms.
24
25 (Choice B) Giant-cell arteritis (temporal arteritis) usually occurs in association w ith polym yalgia rheumatic a
26 and presents w ith systemic symptoms, headache, jaw claudication, visual disturbances, and a significantly
?,7 elevated erythrocyte sedimentation rate (ESR). This patient's absence of these findings makes this less likely.
18
(Choice C) Polyarteritis nodosa typically presents w ith systemic symptoms, skin findings (eg, livedo
20 reticularis, purpura), kidney disease, abdominal pain, and muscle aches or w eakness. ESR is usually
21 elevated, w hich is not seen in this patient.
22
23 (Choice D) Polym yalgia rheumatic a almost alw ays occurs in patients age >50. Patients develop pain and
24 stiffness of the shoulders and pelvic girdle but do not have significant muscle tenderness. ESR is usually
25
26
?.7
18
significantly elevated. Polym yalgia rheumatic a is commonly associated w ith temporal arteritis.

(Choice E) Polym yositis usually presents w ith symmetrical proximal muscle w eakness, mild pain, and
elevated muscle enzymes (eg, creatine kinase, aspartate aminotransferase, lactate dehydrogenase,
aldolase). However, there is usually less prominent hip or shoulder involvement, as seen in this patient. In
I
addition, this patient's absence of muscle w eakness makes polym yositis unlikely.
20
21 (Choice F) Rheumatoid arthritis occurs most commonly in middle-aged w omen. T ypical findings include
22 swelling of the w rist and hand joints (metacarpophalangeal and proximal interphalangeal}, morning stiffness,
23 and elevated rheumatoid factor and ESR.
24
25 (Choice G) Seronegative spondyloarthropathies (SA) commonly cause low back pain . Although the
26 rheumatoid factor is negative, an elevated ESR w ould be expected in any of the SAs. In contrast to
27 fibromyalgia, exercise usually improves pain in SA.
28
Educational objective:
29
Fibromyalgia presents commonly in young-to-middle-aged w omen w ith w idespread pain, fatigue, and
30
cognitive/mood disturbances. Diagnosis is made clinically using w idespread pain index and symptom
31
severity score, w hich emphasize cognitive problems, fatigue, and severity of somatic symptoms.
32
33
34 References:
35 1. The American College of Rheumatology preliminary diagnostic criteria for fibromyalgia
36 and measurement of symptom severity.
37
38
39
Time Spent: 5 seconds Copyright UWorld l ast updated: [10/21120141 v
40
Media Exhibit

Widespread Pain Index & Symptom Severity Score I


Symptom severity scale
Widespread pain index
(0 = no problem, 1 =slight,
(score 0-19)
2 = moderate, 3 = severe)

Neck Fatigue (0-3)


Jaw (left & right) Waking unrefreshed (0-3)
Shoulder (left &right) Cognitive symptoms (0-3)
Upper arm (left & right) Somatic symptoms
Lower arm (left & right) 0 = none
Chest 1 = few
Abdomen 2 = moderate
Upperback 3 = many
Lower back
Hip (left & right) Final score between 0-12

Upper leg (left & right)


Lower leg (left & right)
UWor1d
2
1
" Item: 20 of 44
Q. ld: 3148 [
11 P Mark <J
Previous
[:>
Next
jf
Lab Values
~
Notes
~
Calculator
3
4
5
6 A 58-year-old w oman has had bilateral hip and knee pain for the past several months. The pain is w orse w ith
7 activity and better w ith rest. Recently she has noticed sw elling of the right knee. She reports morning
8 stiffness of about 10-15 minutes duration. She denies fever or w eight loss. Her past medical history is
9 significant for hypertension, type 2 diabetes, gastroesophageal reflux disease, and obstructive sleep apnea .
10 She takes hydrochlorothiazide, lisinopril, omeprazole, insulin and aspirin. She does not use tobacco, alcohol
11 or drugs. Her temperature is 37.2 C (98.9 F), and blood pressure is 146/86 mmHg. Examination show s a
12 mild effusion, tenderness, and decreased range of motion of the right knee. Synovial fluid analysis reveals the
13 follow ing:
14
Cell count 1100/ml
15 Gram stain negative
16 Crystals absent
17
18
Plain films of her knee joint w ould most likely reveal?
19

21 <::, A. Punched out erosions w ith a rim of cortical bone [2%]


22 B. Periarticular osteopenia and joint margin erosions [8%]
23
.; C. Narrowing of joint space and osteophyte formation [82%]
24
25 D. Normal joint space w ith soft tissue sw elling [4%]
26 '.::' E . Calcifications of cartilaginous structures [2%]
?.7
18
19 Explanation: User ld:

21 This patient's presentation is most consistent w ith osteoarthritis (OA). OA is a noninflammatory arthritis that
22 most commonly affects the hands and w eight-bearing joints. Increasing age is the leading risk factor for OA.
23 Patients typically present w ith mild morning stiffness (< 30 minutes) and pain that is exacerbated by exercise
24 and relieved by rest. Unlike autoimmune diseases, systemic symptoms are generally absent. Effusions may
25 occur in OA but the joint w ill remain cool to the touch. Bony crepitus, bony enlargement, and painful or
26 decreased range of motion are also common on exam. Synovial fluid analysis w ill reveal a w hite blood cell
?.7 count of 200-2000/ml (as opposed to 0-200/ml in a normal joint, 2,000-50,000/ml in an inflammatory condition,
18 and >50,000/ml in septic arthritis). Plain films of an osteoarthritic knee w ill reveal a narrow ed joint space w ith
19 osteophyte formation, and subchondral sclerosis/cysts.

(Choice A) Punched out erosions w ith a rim of cortical bone are the characteristic x-ray finding in gouty
arthritis. Gout causes acute joint pain and synovial fluid analysis w ill demonstrate a white blood cell count of
2,000-50,000/ml and needle-shaped, negatively birefringent crystals.

21 (Choice B) The x-ray findings of periarticular osteopenia and joint margin erosions are classic for rheumatoid
22 arthritis (RA). RA is an inflammatory arthritis presenting w ith morning stiffness (> 30 minutes) and systemic
23 symptoms of fever, malaise, and w eight loss. W hite blood cell count of 2,000-50,000/ml on synovial fluid
24 analysis is characteristic. W rists, MCP and PIP joints are commonly involved. The joints w ill be w arm to the
25 touch .
26
27 (Choice D) Normal joint space w ith soft tissue sw elling is seen on radiography in patients w ith infectious
28 arthritis. Cell count of >50,000/ml is typical on synovial fluid analysis and the gram stain is often positive.
29
(Choice E) Calcification of cartilaginous structures (chondrocalcinosis) is typical of pseudogout, or calcium
30
pyrophosphate dihydrate (CPPD) deposition. The typical presentation involves acute sw elling, stiffness, and
31
pain of the knee after surgery or medical illness. Synovial fluid analysis demonstrates a cell count of
32
2,000-50,000/ml w ith rhomboid, positively birefringent crystals.
33
34 Educational objective:
35 Osteoarthritis is a non-inflammatory arthritis presenting w ith pain that is w orse w ith activity and improved w ith
36 rest. X-ray findings include joint space narrow ing and osteophytes. Synovial fluid analysis w ill reveal few er
37 than 2000 WBC/ml, no organisms, and no crystals.
38
39
Time Spent: 4 seconds Copyright UW orld Last updated: [2119/2015]
40 "
2
1
" Item: 21 of 44
Q. ld: 4078 [
11 P Mark <J
Previous
[:>
Next
jf
Lab Values
~
Notes
~
Calculator
3
4
5
6 A 42-year-old male presents to your office complaining of back pain that started two days ago after carrying
7 heavy packages. He denies any weakness or sensory changes in his legs. His past medical history is
8 insignificant. He is not taking any medications and denies drug abuse. His temperature is 36.r C (98 F),
9 blood pressure is 120/76 mmHg, pulse is 80/min, and respirations are 16/min. Physical examination reveals
10 paravertebral tenderness. Lower extremity power is 5/5 and the deep tendon reflexes are 2+. Babinski's sign
11 is negative. Straight-leg raising test is negative at 90 degrees. What is the most probable diagnosis in this
12 patient?
13
14
A. Multiple myeloma [1 %1
15
16 B. Ankylosing spondylitis [1 %1
17 C. Compression fracture of the vertebrae [3%1
18
"' D. Lumbosacral strain [88%1
19
20 E. Herniated disk [4%1
F. Metastatic tumor [0%1
22
G. Increased lumbar lordosis [1 %1
23
24 H. Spondylolisthesis [1 %1
25 I. Epidural abscess [0%1
26
J. Spinal stenosis [1 %1
?.7
18 K. Abdominal aortic aneurysm [0%1
19 L. Cauda equina syndrome [0%1
20
M. Transverse myelitis [0%1
22 N. Vertebral body osteomyelitis [1 %1
23
24
Explanation: User ld:
~?
18 Lumbosacral strain is the most common cause of back pain. It is estimated that the lifetime risk of
19 lumbosacral strain is close to 80%. The clinical scenario described is typical. The pain starts acutely after
20 physical exertion, and it is concentrated in the lumbar area, usually without radiation to the thighs. Physical
examination reveals local tenderness and contraction of the paraspinal muscles. A straight-leg raising test
22 and neurologic examination are typically normal. The treatment includes NSAIDs and early mobilization.
23
24 A herniated disk (Choice E) is characterized by acute pain that radiates to the thighs and typically below the
25 knee. Straight-leg raising test is positive.
26
27 A compression fracture of the vertebrae (Choice C) presents as acute intense pain, and local spinal
28 tenderness is usually observed. Predisposing factors are usually obvious (postmenopausal or senile
29 osteoporosis, steroid treatment).
30
31 Ankylosing spondylitis (Choice B) and multiple myeloma (Choice A) are characterized by chronic back pain.
32
A metastatic or primary tumor (Choice F) is not likely in this case.
33
34 Educational objective:
35 Lumbosacral strain is the most common cause of acute back pain. The typical clinical scenario includes
36 acute onset of the back pain after physical exertion, absence of radiation, presence of paravertebral
37 tenderness, negative straight-leg raising test, and normal neurologic examination.
38
39
Time Spent: 7 seconds Copyright UWorld Last updated: [12/29/20141
40
1
2
" Item: 22 of 44
Q.ld : 3157 [
l ll''Mark <:J
Previous
t>
Next
il
Lab Values
~
Notes
~
Calculator
3
4
5
6 A 66-year-old man comes to the physician complaining of progressive low er back pain. The pain is
7 associated with bilateral leg pain, which is precipitated by w alking. The pain improves with leaning on a cane
8 or sitting. Over-the-counter ibuprofen has provided moderate relief. The patient has no pain at night and no
9 problems with bow el or bladder function. He underwent coronary artery bypass grafting 6 years ago for
10 3-vessel coronary artery disease. Physical examination show s normal strength, reflexes, and sensation in his
11 legs. A straight leg raise test does not reproduce the pain. X-rays of the lumbosacral spine show
12 degenerative changes. Ankle brachial index is 1.1 on the right and 1.2 on the left. Which of the following is
13 most likely responsible for his current condition?
14
15
.~ A. Atherosclerosis [15%]
16
17 B. Disc herniation [4%]
18 .; C. Spinal canal narrowing [63%]
19
20 D. Spinal cord compression [6%]
21 E. Spondyloarthropathy [ 12%]
F. Vertebral metastasis [0%]
23
24
25 Explanation: User ld:
26
?.7
18
19
20
21
Neurogenic & vascular claudication
Neurogenic claudication
(pseudoclaudication)
Vascular claudication
I
23 Posture-dependent pain Exertion ally-dependent pain
24
Lumbar extension worsens Pain relieved with rest,
25
pain (eg, walking downhill) but not with bending forward
26
?,7 Lumbar flexion relieves pain while walking
18 (eg, walking while bent Lower-extremity
19
Symptoms forward) cramping/tightness
20 Lower-extremity numbness No significant lower-extremity
21 & tingling weakness
23 Lower-extremity weakness Possible buttock, thigh, calf,
24 Low back pain or foot pain
25
26 Decreased pulses
?.7 Normal pulses
18 Cool extremities
Examination Frequently nonrnal
19 Decreased hair growth
20 examination
Pallor with leg elevation
21

23 Diagnosis MRI of the spine Ankle brachial index


24
25 @USMLEWorld, LLC
26
?,7 This patient with back and leg pain that is aggravated by spinal extension and relieved with flexion has typical
18 features of lumbar spinal stenosis (SS). SS is due to narrowing of the spinal canal, with resulting
19 compression of lumbar nerve roots. This may result from a number of contributing factors, including
20 degenerative spondylosis, degenerative disk disease, and thickening of the ligamentum flavum. Because SS
21 is primarily a degenerative disorder, patients are generally older and symptoms follow a slow ly progressive
course in most cases.
23
24 The symptoms of SS are posture-dependent. Flexion of the spine causes widening of the spinal canal;
25 extension causes narrowing of the canal. Therefore, the leg pain of lumbar SS is exacerbated by extension
26 of the spine (standing, w alking upright) and improved by flexion (sitting, leaning forward). This phenomenon is
?.7 referred to as "neurogenic claudication" as the patient will experience low er-extremity pain with w alking that
18 may resemble vascular claudication. As opposed to patients with vascular claudication who have pain with
19 exertion and relief on standing still, patients with neurogenic claudication will have pain when w alking upright
20 but relief when w alking while leaning forward (''shopping cart sign"). Diagnosis of SS is confirmed
21 radiologically with MRI of the spine, and treatment may be conservative (eg, physical therapy, home
exercise program) or surgical (eg, laminectomy).
23
24 (Choice A) Atherosclerotic peripheral vascular disease (PVD) often presents with low er leg pain on w alking
25 (intermittent claudication). How ever, the leg pain of vascular claudication is exertional rather than
26 position-dependent and is exacerbated by w alking but not standing still. In addition, PVD does not cause
?,7 associated low back pain . Although this patient's cardiovascular disease history suggests that he likely has
18 peripheral atherosclerosis, his normal ankle brachial index and pulses make neurogenic claudication more
19 likely than PVD-related claudication.
20
(Choice B) Patients with a herniated lumbar disk will typically have pain in the low back that may radiate
21
down the sciatic nerve to the calf and foot (sciatica). The pain may w orsen with prolonged sitting. A positive
straight leg test is sensitive for a herniated disc with sciatica. Sciatica is usually unilateral, as opposed to
23
this patient's bilateral symptoms.
24
25
26
?.7
18
(Choice D) Spinal cord compression presents with back pain, paralysis, hyperreflexia, urinary and fecal
incontinence, or urinary retention.

(Choice E) Spondyloarthropathy refers to a broad spectrum of conditions, of w hich ankylosing spondylitis


I
19 (AS) has the most prominent spinal involvement. AS affects primarily young men and presents most often
20 as low back pain with limited spine mobility. Leg pain is not a prominent complaint. Symptoms of
21 spondyloarthropathy improve with exercise, whereas this patient's symptoms worsen with w alking.

23 (Choice F) Vertebral metastasis should be suspected in patients with low back pain in conjunction with
24 history of malignancy, w eight loss, failure to improve with conservative therapy, and constant dull pain.
25 Symptoms are often nocturnal.
26
Educational objective:
27
Lumbar spinal stenosis is most commonly caused by degenerative changes in the spine and presents with
28
low back and leg pain. The leg pain of neurogenic claudication is position-dependent (exacerbated by lumbar
29
extension) and persists while standing still. Vascular claudication is exertion-dependent and resolves with
30
standing still. Arterial pulses and ankle-brachial index can differentiate between vascular and neurogenic
31
claudication .
32
33
34 References:
35 1. The reliability of differentiating neurogenic claudication from vascular claudication based
36 on symptomatic presentation.
37
38
39
Time Spent: 15 seconds Copyright UW orld Last updated: [11 / 16/2014]
40
1
2
" Item: 23 of 44
Q. ld: 3304 [
11 P Mark <J
Previous
[:>
Next
jf
Lab Values
~
Notes
~
Calculator
3
4
5
6 A 75-year-old w hite male comes to the physician's office for his routine health maintenance examination . He
7 has no symptoms. He has a past medical history significant for hypertension and hyperlipidemia . He takes
8 aspirin, hydrochlorothiazide and simvastatin. He does not smoke and consumes 1-2 beers on w eekends. He
9 w alks 2 miles every morning and eats a balanced diet. His vital signs are w ithin normal limits. His chest is
10 clear to auscultation, and his abdomen is soft and nontender. Rectal examination show s a diffusely enlarged,
11 firm prostate w ithout nodules. Stool for occult blood is negative. The distal interphalangeal joints are
12 enlarged, and his gait is normal. His labs are as follow s:
13 Total bilirubin 1.0 mg/dl
14 Alkaline phosphatase 420 U/L
15 Aspartate aminotransferase (SGOT) 20 U/L
16 Alanine aminotransferase (SGPT) 25 U/L
17 Serum creatinine 0.8 mg/dl
18 Calcium 8.8 mg/dl
19 Serum PSA 2.1 ng/ml
20
21
W hich of the follow ing is the most likely cause of the elevated alkaline phosphatase in this patient?
22

24 A. Metastatic bone disease [ 19%]


25 B. Plasma cell neoplasia [3%]
26
C. Simvastatin [26%]
?.7
18 ~ D. Paget's disease of bone [48%]
19 E. Alcohol use [3%]
20
21
22 Explanation: User ld:

24 Elevation of alkaline phosphatase can result from many conditions. Most commonly, elevations of this
25 enzyme are seen in hepatobiliary disorders or diseases of bone causing increased bone turnover. The single
26 most common cause of asymptomatic isolated elevation of alkaline phosphatase in an elderly patient is
?.7 Paget's disease of bone (osteitis deformans). Paget's disease of bone is most commonly diagnosed in
18 asymptomatic individuals w hen an isolated elevated alkaline phosphatase is incidentally discovered on routine
19 blood tests. Bone lesions in this condition are the result of defective osteoid formation at sites of high bone
20 turnover resulting in hypertrophy of bone. The most commonly affected bones are the skull, clavicles, pelvis
21 and long bones. Pathologic fractures, pain, osteosarcoma and neurologic symptoms are possible
22 complications of this condition .

(Choice A) In patients w ith prostate cancer, prostate-specific antigen (PSA) is usually greater than 4 ng/ml.
24
Prostate cancer w ith bone metastasis may cause elevation of serum alkaline phosphatase but it w ill also
18 typically cause bone pain, osteoblastic bone lesions on radiographic studies and hypercalcemia due to some
19 degree of bone resorption despite the blastic nature of the lesions.
20
21 (Choice B) Patients of multiple m yeloma w ill have bone pain and hypercalcemia due to osteolysis as w ell as
22 monoclonal immunoglobulin proteins in the serum and urine. W eakness, fatigue, w eight loss, renal disease
and amyloidosis are also not uncommonly observed in this condition.
24
25 (Choice C) Statin drugs, such as simvastatin are not know n to cause isolated elevation of serum alkaline
26 phosphatase. They may, how ever, cause an elevation of the aminotransferases (Al T and AST) or
27 rhabdomyolysis leading to m yoglobinuria and renal failure.
28
(Choice E) Alcoholic liver disease causes elevations of the serum alanine aminotransferase (ALT), serum
29
aspartate aminotransferase (AST) and gamma glutamyl transferase (GGT). Elevation of the serum AST is
30
disproportionately higher than the serum ALT and the AST to ALT ratio is typically greater than 2 in liver
31
disease due to alcohol use.
32
33 Educational objective:
34 The most common cause of asymptomatic elevation of alkaline phosphatase in an elderly patient is Paget's
35 disease of bone (osteitis deformans). Patients w ith this condition are typically asymptomatic at the time of
36 diagnosis; the diagnosis is made by incidentally finding an isolated elevated alkaline phosphatase on routine
37 laboratory testing .
38
39
Time Spent: 2 seconds Copyright UW orld Last updated: [2119/2015]
40
1
2
" Item: 24 of 44
Q. ld: 4565 [
11 P Mark <J
Previous
[:>
Next
jf
Lab Values
~
Notes
~
Calculator
3
4
5
6 A 38-year-old male w ith steroid-dependent sarcoidosis presents to the physician's office because of
7 progressive right hip pain . He localizes the pain to the right groin and states that the pain is present on w eight
8 bearing and at rest. His temperature is 37.2 C (98.9 F) and blood pressure is 156/86 mm Hg. Examination
9 show s decreased range of motion due to pain. He also has a round face and fullness in supraclavicular
10 area. Purple striae are present on skin. Muscle pow er is slightly decreased in the proximal thigh muscles in
11 both legs. Reflexes are 2+, and there are no sensory deficits. Plain films of the right hip show no significant
12 abnormalities. ESR is 10 mm/hr. W hich of the follow ing is the most likely cause of his hip pain?
13
14
A. Inflammatory arthritis [5%]
15
16 B . Cartilage degeneration [26%]
17 ~ C. Disruption of bone vasculature [59%]
18
D. Inflammation of the trochanteric bursa [5%]
19
20 E. Aortoiliac occlusion [4%]
21
22
Explanation: User ld:
23
The patient described is most likely experiencing corticosteroid-related osteonecrosis of his right femoral
25 head. Osteonecrosis is also referred to as aseptic necrosis, avascular necrosis, ischemic necrosis and
26 osteochondritis dessicans. There are numerous factors that are believed to play an etiologic role in this
?.7 condition. This process, how ever, proceeds through a common pathw ay regardless of the suspected
18 etiology. In all cases, the vasculature to the affected bone is disrupted causing bone and bone marrow
19 infarction. Subsequently, the bone is unable to remodel and trabecular thinning occurs ultimately leading to
20 collapse of the affected bone. This process may take months to years, on occasion, to occur. Clinically, the
21 patient experiences progressive pain during this time. In the case of osteonecrosis of the femoral head,
22 patients experience anterior hip pain w orsened by activity and relieved by rest w ith progressive limitation of
23 range of motion. Progression of disease w ill ultimately cause joint instability and pain at rest. In the first few
months of symptoms, x-rays w ill often fail to demonstrate an abnormality. MRI is the most sensitive test for
25 this condition. Precipitating factors other than chronic corticosteroid use include chronic alcohol use, trauma
26 and the antiphospholipid syndrome.
?.7
1? (Choice A) There are several forms of inflammatory arthritis including rheumatoid arthritis, gout, pseudogout,
18 psoriatic arthritis, ankylosing spondylitis, lupus and ochronosis among others. Each has a unique
19 presentation, but the case described is most consistent w ith osteonecrosis related to chronic corticosteroid
20 use.
21
22 (Choice B) Cartilage degeneration occurs in osteoarthritis. This condition typically affects patients over 40
23 years of age, though such patients may also present w ith anterior hip pain w orsened by activity. This patient's
clinical scenario is more suggestive of osteonecrosis due to the history of chronic corticosteroid use.
25
26 (Choice D) Inflammation of the trochanteric bursa (trochanteric bursitis) is caused by friction between the
27 tendons of the gluteus medius and tensor fascia lata over the greater trochanter of the femur. Pain is
28 localized over the lateral hip and is w orsened by palpation . Pain caused by pressure on the lateral hip may
29 interfere w ith sleeping in patients w ith this condition.
30
(Choice E) Male patients w ith aortoiliac occlusion (Leriche syndrome) classically present w ith low back, hip,
31
buttock and thigh claudication accompanied by impotence and atrophy of the low er extremities. Femoral
32
pulses are typically w eak and a bruit may be heard over the iliac and femoral arteries.
33
34 Educational objective:
35 Chronic corticosteroid use and chronic excessive ingestion of alcohol account for over 90% of cases of
36 avascular necrosis of bone (osteonecrosis). In the hip, patients present w ith slow ly progressive anterior hip
37 pain w ith limitation of range of motion.
38
39
Time Spent: 2 seconds Copyright UW orld Last updated: [2/19/2015]
40
+ ~
1
2
" Item: 25 of 44
Q. ld: 3155 [
11 P Mark <J
Previous
[:>
Next
jf
Lab Values
~
Notes
~
Calculator
3
4
5
6 A 54-year-old w oman comes to the physician's office complaining of chronic, bilateral knee and hip pain . The
7 pain increases w ith activity and is relieved by rest. She denies fever, chills or w eight loss. The review of
8 systems is unremarkable. She currently w eighs 80 kg (180 lb) and is 146 em (59 in) tall. Her knee joints are
9 tender but there is no w armth, erythema or effusion. X-rays show narrow joint space, subchondral bone
10 cysts, and bony spurs in both knees. W hich of the follow ing interventions w ould provide the greatest long-
11 term benefit to this patient?
12
13
14 A. Muscle strengthening exercise [3%]
15 B. Non steroidal antiinflammatory agents [5%]
16 ~ C. Weight loss [88%]
17
18 D. Chondroitin sulfate [3%]
19 E. Acetaminophen w ith codeine [1 %]
20
21
22 Explanation: User ld:

18 This patient's presentation is most consistent w ith osteoarthritis (OA). Studies have show n a particularly
19 strong link between obesity and knee OA. Based on a height of 59 inches and a w eight of 180 pounds, this
20 patient has a BMI of 36, indicating moderate obesity. Patients w ith OA typically present w ith brief morning
21 stiffness (< 30 minutes) and exertional joint pain. Unlike autoimmune arthritides, systemic symptoms such as
22 fever and w eight loss are generally absent. Plain films of an osteoarthritic knee reveal a narrow ed joint space,
23 osteophytes, and subchondral sclerosis or cysts. Obesity is the most readily modifiable risk factor for
24 OA. W eight loss has been show n to slow progression of the OA and improve joint pain and function. Hence,
w eight loss is the most effective measure in slow ing progression of OA.
26
27 (Choice A) As part of an overall physical therapy program, muscle strengthening exercises play an integral
28 role in OA therapy. They reduce pain and improve mobility, but muscle strengthening does not slow the
29 progression of OA.
30
(Choice B, 0, and E) NSAIDs, acetaminophen, and chondroitin sulfate each play a role in the
31
pharmacotherapy of OA. They provide effective, temporary pain relief but do not slow the progression of OA.
32
33 Educational objective:
34 Plain films of an osteoarthritic knee reveal narrow ed joint space, osteophytes, and subchondral sclerosis or
35 cysts. Obesity is the most readily modifiable risk factor for OA, and w eight loss decreases joint pain,
36 increases function, and slow s progression of the disease. Pharmacologic therapy and exercise also play
37 integral roles in OA therapy.
38
39
Time Spent: 3 seconds Copyright UW orld Last updated: [2119/2015]
40
1
2
" Item: 26 of 44
Q. ld: 4460 [
11 P Mark <J
Previous
[:>
Next
jf
Lab Values
~
Notes
~
Calculator
3
4
5
6 A 72-year-old Caucasian female complains of frequent headaches, fatigue and recent w eight loss. Her
7 shoulder muscles feel stiff in the morning. A tender cord is palpated in the right temporal area . Her ESR is 85
8 mm/hr. W hich of the follow ing is the most likely complication of this patient's condition?
9
10
11 <:: A. Chronic lymphocytic leukemia [12%]
12 B. Hypothyroidism [11 %]
13 .; C. Aortic aneurysm [41 %]
14
15 D. Ulcerative colitis [9%]
16 : E . Membranous glomerulonephritis [26%]
17
18
19 Explanation: User ld:
20
The patient described in the vignette has typical features of giant cell arteritis. Symptoms include headache,
21
jaw claudication, muscle fatigue, and visual disturbance. On examination, patients have scalp tenderness
~?
and a decreased temporal artery pulse. The erythrocyte sedimentation rate (ESR) is generally more than 50
18 mm/hr.
19
20 As giant cell arteritis can involve the branches of aorta, an aortic aneurysm is a w ell-know n complication . For
21 this reason, patients should be follow ed w ith serial chest x-rays.
22
23 (Choice A) Chronic lymphocytic leukemia is the most common form of leukemia in older individuals. The
24 presentation is very insidious. Although the exact etiology is unclear, certain chromosomal abnormalities can
25 predispose individuals to this condition.

27 (Choice B) The most common cause of hypothyroidism is chronic lymphocytic thyroiditis (Hashimoto's
28 disease). Radiation to the thyroid region predisposes to the development of hypothyroidism .
29
30 (Choice D) Ulcerative colitis is an inflammatory bow el disease involving the large intestine. It is more
31 common in Jew s, and those w ith a positive family history.
32
(Choice E) Important causes of membranous glomerulonephritis include hepatitis B and C, syphilis, gold,
33
penicillamine, SLE and rheumatoid arthritis. Many patients have no predisposing factors.
34
35 Educational objective:
36 Aortic aneurysms are w ell-know n complications of giant cell or temporal arteritis due to the involvement of the
37 branches of the aorta . For this reason, patients should be follow ed w ith serial chest x-rays.
38
39
Time Spent: 3 seconds Copyright UW orld Last updated: [1 /21 /2015]
40
1
2
" Item: 27 of 44
Q. ld: 3167 [
11 P Mark <J
Previous
[:>
Next
jf
Lab Values
~
Notes
~
Calculator
3
4
5
6 A 23-year-old w hite w oman presents complaining of 6 months of progressive low back pain. She complains
7 of morning stiffness that lasts one hour, and says her symptoms improve w ith physical activity. She denies
8 rash, eye pain, urinary problems, and diarrhea. Her past medical history is significant only for an
9 appendectomy at the age of 16 years. She has been taking birth control pills for the past l\'l'o years. She is
10 afebrile w ith normal w eight and height. Examination reveals reduced forward flexion of the lumbar spine and
11 tenderness over the sacroiliac joints. The remainder of the physical examination is normal. W hich of the
12 follow ing is the most appropriate next step in the management of this patient?
13
14
15 <::' A. HLA-B27 testing [30%]
16 B. ANA and rheumatoid factor levels [11 %]
17 C. Bone scan [1 %]
18
19 D. MRI of the spine [7%]
20 .; '.::' E . X-ray of the sacro-iliac joints [51%]
21
22
23 Explanation: User ld:
24
Ankylosing spondylitis (AS) is the most likely diagnosis in this patient. The highest incidence of AS occurs in
25
the second and third decades of life, and it has a male:female ratio of approximately 2:1. Any young patient
26
presenting w ith progressive low back pain and spinal stiffness of >3 months duration should be investigated
for this condition . Patients often complain of morning stiffness lasting >30 minutes, and their back pain
18
typically improves w ith exercise. Decreased lumbar spinal mobility and tenderness over the sacroiliac joints
19
are common exam findings. Anteroposterior x-ray of the sacroiliac joints is used to confirm the diagnosis in
20
the setting of a suggestive history and physical. Fusion of the sacroiliac joints and/or bamboo spine is
18 diagnostic.
19
20 (Choice A) There is a strong association bel\'Veen HLA-B27 and ankylosing spondylitis. W hile greater than
21 90% of patients w ith AS have HLA-B27, only 5% of patients w ith HLA-B27 have AS . Therefore, HLA-B27 is not
22 specific for AS and testing for it is not necessary for diagnosis. On the other hand, demonstrating
23 radiographic changes of the lumbar spine is necessary for diagnosis.
24
25 (Choice B) AS belongs to the category of seronegative spondyloarthropathies, so named because patients
26 are negative for rheumatoid factor and ANA.

28 (Choice C) Bone scan is not recommended for diagnosing ankylosing spondylitis. It is best for identifying
29 osteomyelitis, suspected fractures, and neoplasms.
30
(Choice D) W hen AS is strongly suspected clinically and lumbar spine x-ray findings are negative or
31
equivocal, MRI is recommended. MRI is highly sensitive and specific for sacroiliitis and may be diagnostic in
32
such cases. How ever, the effectiveness and low er cost of x-ray make it a logical first step.
33
34 Educational objective:
35 Ankylosing spondylitis is a seronegative spondyloarthropathy that presents w ith progressive inflammatory
36 back pain and stiffness. In the setting of suggestive findings on history and physical, AS is confirmed by plain
37 film x-ray demonstrating fused sacroiliac joints and/or bamboo spine.
38
39
Time Spent: 3 seconds Copyright UW orld Last updated: [211912015]
40
1
2
" Item: 28 of 44
Q.ld: 8895 [
l ll''Mark <:J
Previous
t>
Next
il
Lab Values
~
Notes
~
Calculator
3
4
5
6 A 22-year-old soccer player comes to the emergency department with knee pain . During practice earlier that
7 day, she tried to run around another player on the right and felt a popping sensation in her right knee follow ed
8 by pain and rapid sw elling. The patient has no other injury to the knee. She has had no significant prior knee
9 injuries. Her past medical history is unremarkable. On physical examination, the right knee is significantly
10 sw ollen and tender. Ambulation appears awkward, and the patient hesitates to bear w eight on the right leg.
11 Aspiration of the knee yields grossly bloody joint fluid. Which of the following is the most likely diagnosis?
12
13
~ A Anterior cruciate ligament injury [46%]
14
15 B. Medial collateral ligament injury [14%]
16 C. Meniscal injury [28%]
17
D. Patellofemoral pain syndrome [1 %]
18
19 E. Popliteal cyst rupture [7%]
20 F. Stress fracture of the tibial plateau [3%]
21
22
23 Explanation: User ld:
24
25
26
Features of ant erior cruciate ligament injury
?.7

I
18 Injury Rapid deceleration or direction changes
19
20 mechanisms . Pivoting on lower extremity with foot planted
21
22
23 Pain: rapid onset, may be severe
24 A "popping" sensation at the time of injury
25 Symptoms
Significant swelling (effusion/ h emarthrosis)
26
?,7 Joint instability
18
19
Examination Anterior laxity of tibia relative to femur
20
21 findings (anterior drawer test, Lachman test)
22
23
24 Magnetic resonance imaging
25 Diagnosis
26
?.7 RICE (rest, ice, compression, elevation) measures
18 Treatment
19 +I- Surgery
20
@ USMLEWorld, LLC
21
22
23 Injuries to the anterior cruciate ligament (ACL) are common in young athletes, especially in w omen
24 participating in sports requiring rapid direction changes or pivots on the low er extremity (eg, soccer,
25 basketball, tennis}. ACL tears can also occur in high-contact sports or injuries involving a blow to the knee or
26 significant twisting force. Patients with partial- or full-thickness ACL tears typically experience a "popping"
?,7 sensation in their knee during the injury follow ed by rapid onset of hemarthrosis and a feeling of instability
18 when bearing w eight on the affected side.
19
20 Patients with an ACL tear will show laxity at the knee with the tibia able to be pulled forward relative to the
21 femur. Tw o such maneuvers -the Lachman test and the anterior drawer sign - are highly sensitive
22 (>90%} and specific for ACL injuries. In addition, there is usually a significant knee effusion due to
23 hemarthrosis. The diagnosis is usually confirmed on MRI.
24
(Choice B) Medial collateral ligament (MCL) tears may also occur in pivoting/twisting injuries or if the knee
25
is struck from the lateral side with the foot planted. Examination show s tenderness at the medial knee, and
26
laxity when the foot is gently forced into abduction with the knee stationary (valgus stress test}. How ever,
?.7
MCL injuries are not usually associated with significant hemarthrosis unless there is a concurrent ACL injury.
18
19 (Choice C) Meniscal tears are possible in athletes subjected to rapid direction changes and are
20 characterized by a subacute or chronic locking or popping sensation in the knee. How ever, immediate
21 symptoms at the time of the injury may be mild. Effusions are possible but typically develop slowly, and
22 hemarthrosis is rare.
23
24 (Choice D) Patellofemoral pain is commonly seen in young female athletes with chronic overuse rather
25 than acute trauma to the knee. Patients usually develop pain over the anterior knee that is reproduced by
26 extending the knee while compressing the patella (patellofemoral compression test}. Although the anterior
?,7 knee may have sw elling, true knee effusions and hemarthrosis are not seen.
18
(Choice E) Rupture of a popliteal cyst (Baker cyst} may cause posterior knee pain. How ever, this is more
19
common in older adults and is often associated with osteoarthritis of the knee. Patients may have sw elling at
20
the posterior knee and calf resembling deep venous thrombosis. Knee effusions and hemarthrosis are
21
usually not present.
22
23 (Choice F) Stress fractures of the tibial plateau may occur in female athletes, especially following abrupt
24 initiation or changes in exercise regimen . How ever, stress fractures typically present with progressively
25 w orsening pain and are much less common than ligament injuries.
26
27 Educational objective:
Anterior cruciate ligament (ACL} injuries are common, especially in young athletes in sports requiring rapid
29 direction changes or twisting movements of the low er extremity. ACL injuries usually present with rapid onset
30 of pain and sw elling with hemarthrosis. Examination findings include laxity of anterior motion of the tibia
31 relative to the femur.
32
33 References:
34
35 1. Anterior cruciate ligament rupture: differences between males and females .
36 2. Anterior cruciate ligament injuries: anatomy, physiology, biomechanics, and management.
37
38
39
Time Spent: 3 seconds Copyright UW orld Last updated: [2/ 12/2015]
40
Media Exhibit

ACL tear

ACL tear

Femur

Anterior
cruciate
ligament
Articular
cartilage

Lateral
collateral ....._
ligament Posterior
1 _____. cruciate
Popliteus
tendon
--k"l
y __., ,______..--- 1i gament

-----..__Medial
meniscus
Lateral
meniscus
Transverse
ligament

'-.....Medial ACL
Media Exhibit

Spedal tests for knee examination I


"
Special tests for knee examination

Valgus stress test


Place 1 hand above knee along lateral thigh. Place the
other along medial leg & apply outward pressure along calf

MCULCL Laxity indicates MCL injury


injury Varus stress test
Place 1 hand above knee along medial thigh. Place the
other along lateral leg & apply inward pressure along calf
'-
Laxity indicates LCL injury

Ant erior drawer test


Have patient lie supine with knee flexed at 90 degrees
Grip proximal tibia with both hands & pull anteriorly
Lachman test
ACL
injury Place knee at 30 degrees flexion ~

Stabilize distal femur with 1 hand & pull proximal tibia


anteriorly with the other
Significant anterior displacement of tibia with either test '-
indicates ACL injury

Po sterior drawer test


Flex knee to 90 degrees & stabilize foot
Grasp tibia with both hands & push posteriorly
Significant displacement indicates PCL injury
PCL
injury Posterio r sag test
Place patient supine with hips flexed to 45 degrees, knees
flexed to 90 degrees & feet flat on table
PCL injury causes affected tibia to sag backward relative
to femur ~

Thessaly test
'
Patient stands on 1 leg with knee flexed 20 degrees
Pain or locking with internal &/or external knee rotation
suggests meniscal tear
Apl ey test
~

With patient prone & knee flexed to 90 degrees, stabilize


patient's thigh with examiner's knee or hand
Men iscal
tear Press patient's heel directly toward floor while internally &
externally rotating foot '
Focal pain with compression suggests meniscal tear
McMurray test
Passive knee flexion & extension while placing examiner's
thumb & index finger on medial & lateral joint lines
Clicking with passive movements or medial/lateral rotation
suggests meniscal tear

ACL =anterior cruciate ligamen~ LCL = lateral collateral ligamen~ MCL = medial collateral ligament;
PCL = posterior cruciate ligament.
UWorld
"
~
Media Exhibit

Medial collateral ligament injury

Medial collateral ligament injury

Femur------':-- - -

Direction ---tb~
ofimpact Ill"
Medial
- 7----collateral
ligament

- - - - -Tibia
@ USMLEWortd. LlC
Media Exhibit

Patellofemoral pain syndrome (Runner's knee)

Patellofemoral pain syndrome (PFPS)

Anterior view lateral view

Femur - - - - -

Patellar
- --,--
ligament

) VSM LEWofld. llC


1
2
" Item: 29 of 44
Q. ld: 3165 [
IIPMark <J
Previous
[:>
Next
jf
Lab Values
~
Notes
~
Calculator
3
4
5
6 A 21-year-old w oman presents w ith 4 months of slow ly progressive low back pain . Her back pain is
7 associated w ith early morning stiffness that improves as the day progresses. She has no fever or
8 gastrointestinal complaints. She denies any recent illness. On examination, there is limited range of motion
9 of her back. Other examination is unremarkable. Plain X-ray films show bilateral sacroiliitis. W hich of the
10 follow ing conditions is this patient at greatest risk of developing?
11
12
- A. Aortic coarctation [1 %]
13
14 B. Thoracic aortic aneurysm [4%]
15 C. Renal failure [5%]
16
D. Oral ulcers [4%]
17
18 ~ E . Anterior uveitis [79%]
19 F. Ischemic optic neuropathy [1%]
20
~ G. Episcleritis [4%]
21
22
23 Explanation: User ld:
24
25 Although ankylosing spondylitis (AS) is more common in young males, it is the most likely diagnosis in this
26 patient w ith low back pain and sacroiliitis. The male-to-female ratio of AS is 2-3:1. The highest incidence of
?.7 AS occurs betw een ages 20-30 years, and there is a strong association w ith HLA-B27. AS is primarily a
18 disease of the axial skeleton, causing progressive back pain and spinal stiffness. Morning stiffness and
19 improvement w ith exercise are typical. AS may be definitively diagnosed in a patient w ith symptoms of AS
20 and bilateral sacroiliitis on plain film . Anterior uveitis is the most common extra-articular manifestation of AS
21 and occurs in 25-40% of patients.
22
23 (Choice A) Aortic coarctation is associated w ith Turner's syndrome. Patients w ith AS do not have an
24 increased incidence of aortic coarctation, but they are at risk of aortic regurgitation secondary to scarring of
25 the aortic valve cusps.
26
(Choice B) Thoracic aortic aneurysm is associated w ith Marfan syndrome, Ehlers-Danlos syndrome, and
18 syphilis. It also has an association w ith many inflammatory arthritides, including AS. Therefore, this patient is
19 at some increased risk of developing a thoracic aortic aneurysm. Nonetheless, her odds of developing acute
20 uveitis are far greater.
21
22 (Choice C) Patients w ith AS may develop secondary amyloidosis and lgA nephropathy. How ever, these
23 complications are less common than acute uveitis and only occasionally lead to renal failure.
24
25 (Choice D) Oral ulcers are associated w ith systemic lupus erythematosus, Crohn's disease, and Behcet's
26 disease. How ever, there is not a strong association between AS and development of oral ulcers.
27
28 (Choice F) Ischemic optic neuropathy is a feared complication of temporal arteritis. It results from vasculitis
of the retinal artery and may cause blindness. This condition is not associated w ith AS.
30
(Choice G) Episcleritis is most strongly associated w ith rheumatoid arthritis and inflammatory bow el
31
disease. There is no strong association betw een AS and episcleritis.
32
33 Educational objective:
34 Ankylosing spondylitis (AS) occurs most often in adults aged 20-30 years. Patients suffer from limited spine
35 mobility and progressive back pain of greater than 3 months duration. The pain and stiffness are w orse in the
36 morning and improve w ith exercise. Bilateral sacroiliitis on plain film of a patient w ith the above symptoms is
37 diagnostic. The most prominent extraarticular manifestation of AS is anterior uveitis.
38
39
Time Spent: 3 seconds Copyright UW orld Last updated: [2119/2015]
40
1
2
" Item: 30 of 44
Q. ld: 3150 [
11 P Mark <J
Previous
[:>
Next
jf
Lab Values
~
Notes
~
Calculator
3
4
5
6 A 45-year-old man complains of sudden pain and swelling in his left first metatarsophalangeal joint. He is
7 undergoing high dose induction chemotherapy for acute leukemia. Joint fluid aspiration reveals negative
8 birefringent crystals and elevated w hite cell count. W hich of the follow ing prophylactic measures w ould most
9 likely have prevented this condition?
10
11
12 ") A. Urine acidification [1 %]
13 B. N-acetylcysteine [1 %]
14 .; C. Allopurinol [74%]
15
16 D. Mesna [3%]
17 _ E. Adequate hydration [19%]
18
19
20 Explanation: User ld:
21
This patient presents w ith a classic case of acute gouty arthritis. Gout results from either overproduction or
22
underexcretion of uric acid . In this patient's case, overproduction due to rapid tumor cell lysis is the most
23
likely mechanism. Induction chemotherapy causes rapid tumor destruction, or lysis. This process releases
24
uric acid, a byproduct of DNA, into the circulation and results in an elevated serum uric acid level that can
25
predispose to gout attacks. Acute, monoarticular arthritis affecting the first metatarsophalangeal joint
26
(podagra) is the most common presentation of a gout attack. Definitive diagnosis is achieved by joint
?.7
aspiration w ith demonstration of needle-shaped, negatively birefringent urate crystals and a w hite blood cell
18
count of 2,000-50,000/mm3. Allopurinol, a xanthine oxidase inhibitor, is often used to prevent gout attacks. It
19
w orks by decreasing uric acid production.
20
21 (Choice A) Urine acidification is helpful in eliminating amphetamines but not in eliminating uric acid. In fact,
18 urine alkalinization increases uric acid solubility and decreases uric acid renal stone formation. Acidification
19 w ill increase gouty attacks. Additionally, uricosuric drugs (e.g. probenecid) are used to prevent gout attacks
20 and w ork by increasing urinary excretion of uric acid . There is some risk of inducing uric acid renal stones
21 w ith probenecid, so allopurinol is the first line agent for prevention of gout during chemotherapy.
22
23 (Choice B) N-acetylcysteine is used as an antidote for acetaminophen overdose and as a nephroprotective
24 agent to prevent radiocontrast-induced nephropathy. How ever, it has no role in prevention of gout.
25
26 (Choice D) Mesna is used to prevent hemorrhagic cystitis caused by certain chemotherapeutic agents
27 (cyclophosphamide).
28
(Choice E) Hydration is recommended to decrease the risk of developing uric acid kidney stones. In relation
29
to gout, adequate hydration is recommended during probenecid therapy to prevent the formation of uric acid
kidney stones. Hydration is not sufficient to treat complications of hyperuricemia from chemotherapy,
31
how ever, so allopurinol must be used.
32
33 Educational objective:
34 Gout can result from overproduction or underexcretion of uric acid. Induction chemotherapy results in rapid
35 tumor cell lysis and releases uric acid into the circulation. It is important to distinguish between the prevention
36 and treatment of gout. For prevention, allopurinol and probenecid are used, w hile colchicine, NSAIDs, and
37 steroids are used during acute attacks.
38
39
Time Spent: 5 seconds Copyright UW orld Last updated: [2/ 19/2015]
40
1
2
" Item: 31 of 44
Q.ld: 4585 [
l ll''Mark <:J
Previous
t>
Next
il
Lab Values
~
Notes
~
Calculator
3
4
5
6 A 54-year-old man comes to the physician complaining of morning facial puffiness and bilateral leg sw elling.
7 His other medical problems include recurrent pulmonary infections due to bronchiectasis and psoriasis. His
8 blood pressure is 143/92 mm Hg and pulse is 92/min. His body mass index is 24 kg/m 2. Examination show s
9 a fourth heart sound . Hepatomegaly, palpable kidneys, and 2+ pitting edema of the low er extremities to the
10 knees bilaterally are present on examination. Urinalysis show s 4+ proteinuria and normal urinary sediment.
11 W hich of the follow ing is the most likely diagnosis in this patient?
12
13
14 ~ t:_ A. Amyloidosis [48%]
15 B. Hepatorenal syndrome [14%]
16 C. Hypertensive nephrosclerosis [4%]
17
18 D. lgA nephropathy [15%]
19 _ E. Polycystic renal disease [19%]
20
21
22 Explanation: User ld:
23
24
25 Cl inical features of secondary amyl oidosis
26
?.7 . Extracellular deposit of insoluble polymeric protein fibrils
18
19 in tissues & organs, elevated amyloid A
20 . Can be secondary to chronic inflammatory conditions
21 . Inflammatory arthr itis (eg, rheumatoid arthritis)
22
23 Epidemiology . Chron ic inf ection s (eg, bronchiectasis, tuberculosis,
24 osteomyelitis)
25 . Inflammatory bowel di sease (eg, Crohn's disease)
26
?,7 . Malign an cy (eg, lymphoma)
18 . Vasculit is
19
20
21
. Asymptomatic proteinuria or nephrotic syndrome
22 . Cardiomyopathy with heart failure
23 . Hepatomegaly
24
25 Clinical . Mixed sensory & motor peripheral neuropathy &/or
26 p r esentation autonomic neuropathy
?.7 . Visible organ enlargement (eg, macroglossia)
18
19
. Bleeding diathesis
20 . Waxy thickening, easy bruising of skin
21
22
23 Diagnosis . Abdominal fat pad aspiration biopsy
24
25
26
. Treatment of underlying condition
?,7
Treatment
. Colchicine for prevention & treatment
18
>U9JILEWorld, LLC
19
20
This patient's clinical presentation of facial sw elling, bilateral low er-extremity edema , and massive proteinuria
21
are consistent w ith nephrotic syndrome. The presence of nephrotic syndrome w ith palpable kidneys,
22
hepatomegaly, and ventricular hypertrophy (suggested by an audible fourth heart sound) in the setting of
23
chronic inflammatory disease (recurrent pulmonary infections, bronchiectasis) is suggestive of secondary
24
amyloidosis (AA) as a common unifying diagnosis. Treatment is usually directed at the underlying
25
inflammatory disease. Colchicine is approved for both the treatment and prophylaxis of AA.
26
?.7 (Choice B) Hepatorenal syndrome refers to acute renal failure in patients w ith acute or chronic liver disease
18 and portal hypertension . It usually presents in patients w ith severe liver disease w ith progressively rising
19 serum creatinine w ith minimal proteinuria (<500 mg/day).
20
21 (Choice C) Hypertensive nephrosclerosis is seen in patients w ith chronic hypertension and is associated
18 w ith retinopathy, left ventricular hypertrophy, progressive renal failure, and mild proteinuria (generally <1
19 g/day). Kidneys are usually small, and hepatomegaly is not present.
20
21 (Choice D) lgA nephropathy is a type of nephritis that typically presents w ith hematuria on urinalysis. Unlike
22 in this patient, lgA nephropathy is usually restricted to the kidneys.
23
24 (Choice E) Patients w ith polycystic kidney disease typically present w ith flank pain, hematuria, renal failure,
25 hypertension, and large palpable kidneys. Hepatomegaly can also be present due to co-occurrence of hepatic
26 cysts in some patients. How ever, they usually do not have the proteinuria or generalized edema seen in this
27 patient.
28
Educational objective:
29
Secondary amyloidosis is a complication of a chronic inflammatory condition (eg, chronic infections,
30
inflammatory bow el disease, rheumatoid arthritis) resulting in extracellular tissue deposition of fibrils into
tissues and organs. Patients can develop multi-organ dysfunction (eg, kidneys, liver, gastrointestinal tract).
32
Treatment usually involves treating the underlying inflammatory disease.
33
34
35 References:
36 1. AA amyloidosis: basic knowledge, unmet needs and futur e treatments.
37
38
39
Time Spent: 3 seconds Copyright UW orld Last updated: (9/3/2014]
40
2
1
" Item: 32 of 44
Q. ld: 3898 [
11 P Mark <J
Previous
[:>
Next
jf
Lab Values
~
Notes
~
Calculator
3
4
5
6 A 25-year-old w oman complains of joint pain, fatigue, ankle edema and facial rash . Urinalysis revealed
7 moderate proteinuria. Serum C3 level is decreased. W hat is the most probable mechanism of kidney damage
8 in this patient?
9
10
A. lgE-mediated reaction [1 %]
11
12 ~ B . Immune complex-mediated damage [89%]
13 C. Cytotoxic antibodies [6%]
14
D. Delayed hypersensitivity reaction [1%]
15
16 E. Non-immune damage [2%]
17
18
Explanation: User ld:
19
20 This clinical vignette describes classical presentation of systemic lupus erythematosus (SLE): young w oman
~! w ith joint pain, fatigue, ankle edema and facial rash. Although SLE can cause several types of
18 immune-mediated tissue damage and at the same time causes several w ell defined kidney diseases, it is
19 believed that the main mechanism of kidney damage in SLE patients is immune complex-mediated . Immune
20 complexes circulate in the blood and are deposited in renal glomeruli causing their damage via several
21 mechanisms. One of these mechanisms includes complement activation. That's w hy serum C3 level is
22 decreased in immune complex-mediated kidney diseases such as SLE and poststreptococcal
23 glomerulonephritis.
24
25 lgE-mediated reaction (choice A) is important in allergic interstitial nephritis like methicillin-induced interstitial
26 nephritis.
27
28 Cytotoxic antibodies (choice C) cause Goodpasture's syndrome.
29
30 Delayed hypersensitivity reaction (choice D) may be important in non-specific chronic glomerulonephritis.
31
Non-immune kidney damage (choice E) may be related to toxins, hypertension, diabetes mellitus or other
factors.
33
34 Educational objective:
35 Although SLE can cause several types of immune-mediated tissue damage and at the same time causes
36 several w ell defined kidney diseases, it is believed that the main mechanism of kidney damage in SLE
37 patients is immune complex-mediated.
38
39
Time Spent: 3 seconds Copyright UW orld Last updated: [2/ 19/2015]
40 ~.
---- - -.- - - - - - - - - - - - - -
1
2
" Item: 33 of 44
Q. ld: 3203 [
11 P Mark <J
Previous
[:>
Next
jf
Lab Values
~
Notes
~
Calculator
3
4
5
6 A 32-year-old woman comes to the physician with pain and paresthesias in her right thumb, first two fingers,
7 and the radial half of her ring finger. Her pain is worse at night and interferes with sleep. She also has some
8 numbness in her left hand, but it is less bothersome. The patient feels tired and has lately been unable to
9 function effectively at work. She takes over-the-counter laxatives for constipation. She also complains of
10 heavy menstrual flow. Vital signs are within normal limits. Her skin is dry. When the patient is asked to
11 maintain acute wrist flexion for 30 seconds, she experiences severe pain and paresthesias. Which of the
12 following is the most likely cause of her hand symptoms?
13
14
A. Accumulation of fluid in the carpal tunnel [24%]
15
16 "' B. Accumulation of matrix substances [45%]
17 C. Amyloid fibral deposition [5%]
18
D. Synovial tendon hyperplasia [5%]
19
20 E. Tenosynovial inflammation [21 %]
21
22
Explanation: User ld:
23
24 The patient has nocturnal symptoms of pain and paresthesia in the right median nerve distribution that are
25 worse with flexion of the wrist (positive Phalen test). This is consistent with a diagnosis of carpal tunnel
26 syndrome (CTS) with compression of the median nerve within the carpal tunnel in the wrist. Her symptoms of
?.7 fatigue, constipation, menorrhagia, and dry skin are all suggestive of hypothyroidism. Carpal tunnel syndrome
18 occurs frequently in those with hypothyroidism, in up to 30% of patients compared to 3%-5% in the general
19 population.
20
21 The pathogenesis of CTS is thought to be deposition of mucopolysaccharide protein complexes within the
22 perineurium and endoneurium of the median nerve and in the tendons and synovial sheath that causes direct
23 pressure on the median nerve within the carpal tunnel. CTS in hypothyroidism patients is commonly bilateral
24 and presents with more severe symptoms compared to other etiologies. The pathologic findings and
25 symptoms of CTS often improve with thyroid hormone replacement therapy. All patients with CTS, especially
26 bilateral, should be screened with a detailed history, physical examination, and serum thyroid-stimulating
?.7 hormone levels for the presence of hypothyroidism .
18
19 (Choice A) Accumulation of fluid in the carpal tunnel can cause CTS in pregnancy, particularly in the third
20 trimester.
21
22 (Choice C) Amyloid fibril deposition within the carpal tunnel causes CTS in patients with primary systemic
23 amyloidosis as well as those with secondary forms. The most common CTS due to amyloid deposition
18 occurs in the setting of end-stage renal disease or chronic hemodialysis from deposition of beta 2
19 microglobulin.
20
(Choice D) Patients with acromegaly frequently have an associated bilateral median nerve compression and
21
CTS due to soft-tissue enlargement caused by synovial edema and tendon hyperplasia.
22
23 (Choice E) Patients with rheumatoid arthritis have inflammation of the tendons and synovial sheath. This in
24 turn causes increased compartmental pressure in the carpal tunnel and can lead to CTS.
25
26 Educational objective:
27 Carpal tunnel syndrome occurs in approximately 30% of patients with hypothyroidism. Deposition of
28 mucopolysaccharide protein complexes within the perineurium and endoneurium of the median nerve is
29 thought to be primarily responsible for its pathogenesis.
30
31 References:
32
1. Prevalence of smoking, obesity, diabetes mellitus, and thyroid disease in patients with
34 carpal tunnel syndrome.
35 2. Neuromuscularfindings in thyroid dysfunction: a prospective clinical and
36 electrodiagnostic study.
37
38
39
Time Spent: 2 seconds Copyright UWorld Last updated: [2124/2015]
40 "
1
2
" Item: 34 of 44
Q. ld: 3209 [
IIPMark <J
Previous
[:>
Next
jf
Lab Values
~
Notes
~
Calculator
3
4
5
6 A 52-year-old w oman presents w ith gradual-onset w eakness in her leg muscles. She has noted difficulty
7 climbing stairs and raising from a chair that has been w orsening for the past few years. She initially related
8 her symptoms to simply being "out of shape." She has no problem w ith chew ing food. She has no pain in her
9 muscles and has no skin rash. She takes metoprolol and hydrochlorothiazide for hypertension . She does not
10 use tobacco, alcohol or drugs. Her vital signs are w ithin normal limits. Examination reveals mild w eakness of
11 the thigh muscles in both legs. Deep tendon reflexes and sensory examination show s no abnormalities.
12 Babinski sign is negative. W hich of the follow ing is most likely abnormal in this patient?
13
14
15 A. MRI of the spine [9%1
16 B. Electroencephalogram [1 %1
17 C. Temporal artery biopsy [5%1
18
19 D. Lumbar puncture [2%1
20 .; E . Muscle biopsy [72%1
21 F. Visual evoked potentials [5%1
22
23 G. Electronystagmogram [3%1
24 H. Bone scan [2%1
25 I. Bone marrow biopsy [0%1
26
?.7
18 Explanation: User ld:
19
20 Polym yositis (PM) is an inflammatory muscle disease of unknow n etiology. It presents initially w ith slow ly
21 progressive proximal w eakness of the low er extremities characterized by difficulty ascending and descending
22 stairs or rising from a seated position. The w eakness usually progresses over the course of years; an acute
23 presentation is not typical. Proximal arm w eakness usually follows leading to complaints of difficulty w ith
24 combing hair or w orking w ith the hands overhead. There are no skin findings in PM, w hich helps to distinguish
25 it from dermatomyositis. In PM, the muscles of mastication and facial expression are classically spared;
26 how ever, patients w ith polym yositis may develop dysphagia due to involvement of the striated muscles of the
?.7 upper pharynx . W eakness is the primary complaint in patients w ith PM, and w hile muscle tenderness may be
18 present, severe tenderness or tenderness in the absence of w eakness should lead to the consideration of
other diagnoses such as polym yalgia rheumatic a. A muscle biopsy is the best diagnostic study for
18 polym yositis. MRI of the affected muscles can assist in identifying the best location for a muscle biopsy.
19 Biopsy in polym yositis show s a mononuclear infiltrate surrounding necrotic and regenerating muscle fibers. In
20 more advanced disease, an inflammatory infiltrate may be absent and replacement of muscle w ith fibrotic
21 tissue may be evident.
22
23 (Choice A) MRI of the spine is useful in identifying conditions that may cause compression of the spinal cord
24 or spinal nerve roots such as herniated discs or spinal stenosis.
25
26 (Choice B) Electroencephalography (EEG) is used primarily in the evaluation of patients w ith seizure or sleep
27 disorders.
28
29 (Choice C) A temporal artery biopsy is necessary to make the diagnosis of temporal arteritis.
30
(Choice D) Lumbar puncture is useful in diagnosing meningitis, subarachnoid hemorrhage and inflammatory
31
conditions of the CNS such as multiple sclerosis.
32
33 Educational objective:
Polym yositis is an inflammatory muscle disease of unknow n etiology that presents initially w ith slow ly
35 progressive proximal w eakness of the low er extremities characterized by difficulty w ith stair climbing or rising
36 from a seated position. Muscle tenderness also occurs in some patients. A muscle biopsy is the best
37 diagnostic study for polym yositis.
38
39
Time Spent: 7 seconds Copyright UW orld Last updated: [2119/20151
40
..
1
2
" Item: 35 of 44
Q. ld: 4264 [
11 P Mark <J
Previous
[:>
Next
jf
Lab Values
~
Notes
~
Calculator
3
4
5
6 A 43-year-old construction w orker presents to your office 2 w eeks after an episode of acute back pain . The
7 pain started after lifting a heavy box, w as localized to the low er back w ith little radiation to the buttocks, and
8 quickly responded to conservative treatment. His past medical history is insignificant. He smokes 2 packs a
9 day and consumes alcohol occasionally. He is concerned about the possibility of recurrence of the pain .
10 W hich of the follow ing w ould be most helpful in preventing another episode of back pain in this patient?
11
12
13 0 A. Limiting the physical activity as much as possible [5%]
14 B. Sleep on the stomach [1 %]
15 C. Regular exercise w ith repetitive twisting and bending [11 %]
16
17 D. Bend at the w aist, not at the knees [3%]
18 .; E. Keep the back straight w hile lifting an object [79%]
19 F. Avoid w arm-up exercises before sporting activities [1 %]
20
21
22 Explanation: User ld:
23
24 This patient presents w ith an episode of acute back pain that seems to be caused by lumbar strain (relation to
25 lifting a heavy object, no radicular signs, and good response to conservative therapy). Proper patient
26 education is very helpful in protecting the back from recurrent injury. The education should emphasize the
27 importance of strengthening the supporting muscles (including abdominal muscles) by regular exercise
28 (choice A), choosing an appropriate sleeping posture (e.g. avoiding sleeping on the stomach) (choice B),
29 and learning proper techniques for bending and lifting objects. Exercises w ith repetitive twisting and bending
30 should be avoided (choice C). It is important to bend at the knees, not at the w aist (choice D). W hile lifting an
31 object, one should also bend the knees, keeping the back straight; this technique is very useful in preventing
32 strains and back injuries. W arm-up exercises should be done before any sporting activities (choice F).
33
34 Educational Objective:
Proper patient education (e.g. teaching the proper techniques for bending and lifting objects) is very helpful in
36 protecting the back from recurrent injury.
37
38 Time Spent: 5 seconds Copyright UW orld Last updated: [2/ 19/2015]
39
40
.. .
1
2
" Item: 36 of 44
Q. ld: 4369 [
11 P Mark <J
Previous
[:>
Next
jf
Lab Values
~
Notes
~
Calculator
3
4
5
6 A 65-year-old man complains of periodic back pain radiating to his thigh and buttock. The pain is related to
7 w alking or climbing the stairs but is promptly relieved by leaning forward . He also has noticed tingling and
8 numbness in both low er extremities. He has a history of hypertension and takes hydrochlorothiazide. He
9 does not use tobacco, alcohol, or illicit drugs. His pulse is 76/min, respirations are 14/min, and blood
10 pressure is 140/80 mmHg. Lumbar extension reproduces the pain and tingling, w hile lumbar flexion relieves
11 the symptoms. W hich of the follow ing is the most likely cause of this patient's condition?
12
13
14 A. Iliac artery atherosclerosis [2%1
15 B. Abdominal aortic aneurysm [1 %1
16 ~ C. Degenerative central canal stenosis [69%1
17
18 D. Lumbar disk herniation [23%1
19 E. Spina bifida occulta [1 %1
20 F. Metastatic disease [ 1%1
21
22 G. Seronegative spondylarthritis [3% 1
23
24 Explanation: User ld:
25
26 Lumbar spinal stenosis is a degenerative condition w here the spinal canal is narrow ed, resulting in
?.7 compression of one or more of the spinal roots. It most commonly results from a combination of two factors:
18 enlarging osteophytes at the facet joints and hypertrophy of the ligamentum flavum. Most affected patients are
19 over age 60. Patients experience back pain that radiates to the buttocks and thighs. Numbness and
20 paresthesias may occur. Symptoms are typically w orse during w alking and lumbar extension, w hile lumbar
21 flexion alleviates the symptoms. Pedal pulses should remain normal, as arterial perfusion is not affected.
22 The diagnosis is confirmed w ith MRI.
23
24 (Choice A) Iliac artery atherosclerosis causes claudication in the buttocks and thighs. This pain typically
25 occurs during activity as is relieved by rest. The key clinical differences between iliac artery atherosclerosis
26 and lumbar spinal stenosis are that the former is not affected by lumbar extension or flexion and does not
?.7 cause back pain .

18 (Choice B) Abdominal aortic aneurysms (AAA) are usually asymptomatic but can present w ith severe back or
19 abdominal pain and hypotension upon rupture.
20
21 (Choice D) Lumbar disk herniation typically presents w ith the acute onset of back pain w ith or w ithout
22 radiation dow n one leg. Patients usually recall an inciting event. As w ith lumbar spinal stenosis, the pain may
23 be radiating and be associated w ith neurologic symptoms. How ever, in disc herniation, lumbar flexion and
24 sitting w ill make the pain w orse.
25
26 (Choice E) Spina bifida occulta is a congenital defect of the vertebral arch that does not involve protrusion of
27 the cord or dura. It is typically asymptomatic and diagnosed as an incidental radiographic finding.
28
29 (Choice F) Metastatic disease of the vertebrae typically presents as pain that is chronic, dull, w orse at night,
30 and changes little w ith activity. The pain is usually non-radiating.
31
(Choice G) Seronegative spondylarthritis (e.g., ankylosing spondylitis) is characterized by progressive
32
limitation of back motion. It most often occurs in young men. Back pain and stiffness are typically w orst in the
33
morning and improve as the day progresses.
34
35 Educational Objective:
Lumbar spinal stenosis is a common cause of back pain in patients over 60 years of age. It is characterized
37 by back pain radiating to the buttocks and thighs that interferes w ith w alking and lumbar extension.
38
39
Time Spent: 3 seconds Copyright UW orld Last updated: [2/19/20151
40
1
2
" Item: 37 of 44
Q. ld: 3166 [
11 P Mark <J
Previous
[:>
Next
jf
Lab Values
~
Notes
~
Calculator
3
4
5
6 A 23-year-old man complains of progressive low back pain for the past several months. He has morning
7 stiffness that lasts for one hour. He has no history of trauma to his back and denies any recent illness. He
8 had an episode of pain, blurring, and photophobia of his right eye two years ago that required ophthalmology
9 evaluation . His vital signs are w ithin normal limits. Examination show s tenderness in both sacroiliac joints
10 and reduced range of motion of the low er back. If measured, w hich of the follow ing is most likely to be
11 present in this patient?
12
13
14 A. Rheumatoid factor [2%]
15 B. Anti-cyclic citrullinated peptide (CCP} antibodies [2%]
16 C. Anti dsDNA antibodies [1 %]
17
18 v D. HLA B 27 positivity [93%]
19 E. Glutamic acid decarboxylase antibodies [1 %]
20 F. Acetylcholine receptor antibodies [1 %]
21
22
23 Explanation: User ld:
24
25 Ankylosing spondylitis (AS} is most common in men aged 20-30 years, and presents w ith progressive back
26 pain and morning stiffness of greater than 3 months duration. Although there are no pathognomonic signs of
?.7 AS on exam, reduced range of motion in the low er back and sacroiliac tenderness support the diagnosis. In a
18 patient w ith suggestive signs and symptoms, the diagnosis may be confirmed by a plain film demonstrating
sacroiliitis. In addition to skeletal involvement, patients w ith AS may develop extraarticular manifestations.
18 Anterior uveitis is the most common extraarticular manifestation, occurring in 25- 40% of patients. It presents
19 w ith monocular pain, blurring and photophobia . There is a strong correlation between HLA-827 and AS, w ith
20 HLA-827 positivity occurring in 90% of all patients afflicted by the disease.
21
22 (Choices A and B) Positive rheumatoid factor and anti-CCP antibodies are typically seen in rheumatoid
23 arthritis. AS belongs to the seronegative spondyloarthropathies, so named because they test negative for
24 these serologic markers.
25
26 (Choice C) Anti-dsDNA antibodies are highly specific for systemic lupus erythematosus. This disease may
27 present w ith arthritis, rash, renal disease, and systemic symptoms, but sacroiliac inflammation is uncommon.
28
29 (Choice E) Glutamic acid decarboxylase antibodies are seen in 70% of type I diabetics at the time of
30 diagnosis.
31
(Choice F) Acetylcholine receptor antibodies are seen in m yasthenia gravis.
32
33 Educational objective:
34 Ankylosing spondylitis (AS} is a seronegative spondyloarthropathy associated w ith HLA-827. It is most
35 common in young men and presents w ith progressive, inflammatory back pain and stiffness. Reduced low er
36 back range of motion and sacroiliac tenderness are typical on exam. The most common extraarticular
manifestation is anterior uveitis.
38
39
Time Spent: 3 seconds Copyright UW orld Last updated: [12/29/2014]
40
1
2
" Item: 38 of 44
Q.ld : 3318 [
l ll''Mark <:J
Previous
t>
Next
il
Lab Values
~
Notes
~
Calculator
3
4
5
6 A 35-year-old w oman comes to the physician with pain and stiffness of her w rists and hand joints for the last
7 several months. Her morning stiffness lasts for more than an hour. She also complains of joint sw elling. Her
8 past medical history is significant only for a similar episode a year ago, which resolved with over-the-counter
9 ibuprofen. Joint examination show s mild redness, w armth, sw elling, and tenderness in the proximal
10 interphalangeal and metacarpophalangeal joints and wrists. X-rays show periarticular osteopenia and
11 erosions of the proximal interphalangeal and metacarpophalangeal joints. The patient began taking
12 indomethacin, which provides good relief, but symptoms recur if she skips a dose. Which of the following is
13 the most appropriate next step in management of this patient?
14
15
16 A Azathioprine [1 %1
17 B. Celecoxib [3%1
18 C. Etanercept [2%1
19
20 D. Glucocorticoids [9%1
21 E. lnfliximab [3%1
22 .; F. Methotrexate [70%1
23
24 G. No additional treatment [11 %1
25
26 Explanation: User ld:
?.7
18
19
20
21
22
This patient's presentation is consistent with moderate-to-severe rheumatoid arthritis (RA). RA is a chronic,
systemic inflammatory disorder with progressive erosion of the bone and cartilage and significant joint
destruction and deformity. Patients typically present with joint findings (eg, pain, stiffness, swelling), morning
stiffness, involvement of proximal interphalangeal and metacarpophalangeal joints, and radiographic evidence
I
of erosions and/or periarticular osteopenia . Treatment goals in RA are to induce and maintain early
23 remission, control synovitis, and prevent progression of joint damage.
24
25
26 Rheumatoid arthritis
?,7
18
19
20
21 Methotrexate
22
23
24
25
26 Persistent symptoms for> 6 months
?.7
18 I
19
20
21
+ Inadequate
Step-up therapy Parallel therapy
22 response
23 Add b iologic agent Add another non-biologic agent
24 (e.g., TNF inhibitor) (e.g., sulfasalazine, hydroxychloroquine)
25
26
?,7
18
19 Inadequate response
20
21
22
23
24 Switch to alternative TNF inhibitor
25
& continue methotrexate
26
?.7
18 <;) USMLEWorld, l lC

19
20 All patients diagnosed with RA should be started on disease-modifying antirheumatic agents (DMARDs) as
21 soon as possible as joint damage begins early in its course (Choice G). Nonsteroidal anti-inflammatory
22 drugs and COX-2 inhibitors (eg, celecoxib) are adjunctive therapies for symptomatic relief but do not reduce
23 disease progression. Glucocorticoids can also relieve symptoms and short-term radiographic progression
24 but are also not effective in preventing eventual joint destruction. In fact, they can result in generalized bone
25 loss (ie, osteoporosis).
26
?,7 DMARDs include nonbiologic agents (eg, methotrexate, hydroxychloroquine, sulfasalazine, leflunomide,
18 azathioprine) and biologic agents (eg, etanercept, infliximab, adalimumab, tocilizumab, rituximab).
19 Methotrexate is the preferred initial DMARD in patients with moderately to severely active RA due to its
20 efficacy and long-term safety profile. Patients should be tested for hepatitis B and C and tuberculosis
21 before starting therapy. Methotrexate should not be used in patients who are pregnant or are planning to
22 become pregnant in the near future and those with severe renal insufficiency, liver disease, or excessive
~9
alcohol intake. Patients who do not respond after 6 months may require biologic DMARDs such as tumor
18 necrosis factor-alpha inhibitors (eg, etanercept, infliximab) as step-up therapy (Choices C and E).
19
(Choice A) Azathioprine is an immunosuppressive agent that has been used to treat RA. How ever, it is not
20
as efficacious as other DMARDs and is associated with significant hematologic and gastrointestinal toxicity.
21
22 (Choices B and D) This patient notes improvement with indomethacin and does not require additional
23 symptomatic treatment with celecoxib or glucocorticoids.
24
25 Educational objective:
26 All rheumatoid arthritis (RA) patients should receive disease-modifying antirheumatic drugs (DMARDs) as
27 early as possible in the disease course. Methotrexate is the initial DMARD of choice in most patients with
28 active RA. Nonsteroidal anti-inflammatory drugs or glucocorticoids should be used for initial temporary
29 symptomatic relief while aw aiting response to DMARD therapy.
30
31 References:
32
33 1. In the clinic. rheumatoid arthritis.
34 2. 2012 update of the 2008 American College of Rheumatology recommendations for the use
35 of disease-modifying antirheumatic drugs and biologic agents in the treatment of
36 rheumatoid arthritis.
37

Time Spent: 3 seconds Copyright UW orld Last updated: [9/6/20141 v


.
. . .. . . . ' . . . .
1
2
" Item: 39 of 44
Q. ld: 4205 [
11 P Mark <J
Previous
[:>
Next
jf
Lab Values
~
Notes
~
Calculator
3
4
5
6 A 60-year-old male presents to your office complaining of right-sided neck pain and numbness over the
7 posterior surface of the forearm. He had several such episodes over the last two years that responded to
8 NSAIDs and physical therapy. Physical examination reveals limited neck rotation and lateral bending. There is
9 decreased pinprick sensation on the posterior aspect of the right forearm, but no muscle w eakness is
10 present. Triceps reflex is normal. W hich of the follow ing is the most likely finding on the neck radiography of
11 this patient?
12
13
14 A. Reversed lordotic curve [7%1
15 v B. Bony spurs [64%1
16 C. Vertebral body osteoporosis [1 0%1
17
18 D. Osteolytic lesions (3%1
19 E. Compression fracture [ 15%1
20
21
22 Explanation: User ld:
23
This patient is most likely suffering from cervical spondylosis. It is estimated that cervical spondylosis affects
24
10% of people older than 50 years of age. The history of chronic neck pain is typical. Limited neck rotation and
25
lateral bending is due to osteoarthritis and secondary muscle spasm . Sensory deficit is due to
26
osteophyte-induced radiculopathy and isolated sensory abnormalities are associated w ith good prognosis.
27
Typical radiographic findings include bony spurs and sclerotic facet joints. Interestingly, such 'osteoarthritic'
28
changes are common in asymptomatic patients older than 50 years of age; therefore, specificity of these
29
findings are low . Other findings during cervical spondylosis may include narrow ing of the disk spaces and
30
hypertrophic vertebral bodies (Choice C).
31
32 Reversed lordotic curve (Choice A) is characteristic for severe cervical strain . Osteolytic lesions (Choice D)
33 and compression fracture (Choice E) are unlikely in this case.
34
35 Educational Objective:
36 Osteophytes are the most common findings on cervical radiography in patients w ith cervical spondylosis, but
37 specificity of these findings is low .
38
Time Spent: 2 seconds Copyright UW orld Last updated: (1 2/29/20141
40
1
2
" Item: 40 of 44
Q. ld: 3312 [
11 P Mark <J
Previous
[:>
Next
jf
Lab Values
~
Notes
~
Calculator
3
4
5
6 A 30-year-old w hite male presented to your office w ith low back pain and stiffness. His pain is w orse in the
7 morning and is improved w ith activity. He has also been having bloody diarrhea for the past few days. On
8 examination, he has painful erythematous nodules over his shins. Pain and stiffness is present in his low er
9 back. Plain radiographs show sacroiliac joint inflammation . Stool cultures are negative. Laboratory studies
10 show anemia and thrombocytosis. P-ANCA is positive in high titers. W hich of the follow ing is the most likely
11 cause of his symptoms?
12
13
14 A. Reactive arthritis from diarrhea [22%1
15 v B. lnfiammatory bow el disease [64%1
16 C. Infection w ith Tropheryma Wli,opelii[l%1
17
18 D. Gluten-sensitive enteropathy [5%1
19 E. Infection w ith Giardia Iambiia [ 1%1
20
21
22 Explanation: User ld:
23
This patient described is most likely suffering from inflammatory bow el disease (lBO), specifically ulcerative
24
colitis (UC), given his young age, history of acute-onset bloody diarrhea, anemia and negative stool cultures.
25
lBO typically presents in patients during their second or third decade, and the presenting symptoms of UC
26
often include bloody diarrhea, low er abdominal pain and tenesmus. Common extracolonic manifestations
?.7
include skin findings, such as erythema nodosum (as described in this patient) and pyoderma gangrenosum,
18
episcleritis, arthritis and cholangitis. p-ANCA is positive in ulcerative colitis. Presentation of arthritis is similar
18 to ankylosing spondylitis.
19
20 (Choice A) Reactive arthritis may follow an infectious diarrhea caused by Shigella, Salmonella, Yersinia,
21 Campylobacteror C. difficile. Reactive arthritis is characterized by urethritis, conjunctivitis I uveitis, arthritis,
22 malaise and characteristic cutaneous findings (keratoderma blennorrhagica and balanitis circinata).
23
24 (Choice C) Infection w ith Tropheryma Wli,opeliicauses W hipple disease, a protean disease characterized
25 most commonly by chronic malabsorptive diarrhea, w eight loss, migratory non-deforming arthritis,
26 lymphadenopathy and a low -grade fever.
27
28 (Choice D) Gluten-sensitive enteropathy (celiac disease, celiac sprue) is an autoimmune disease
29 characterized by malabsorptive diarrhea. Serologic testing show s anti-endomysia! and anti-transglutaminase
30 antibodies. The characteristic cutaneous finding is dermatitis herpetiformis.
31
(Choice E) Infection by the protozoan Giardia Iambiia typicaiiy causes a frothy, foul-smelling steatorrhea due
32
to malabsorption caused by the infecting organism . Bloody diarrhea is uncommon.
33
34 Educational objective:
35 Inflammatory bow el disease may occur in association w ith an inflammatory arthritis. Ankylosing spondylitis
36 and lBO are both associated w ith HLA-B27 and may occur in association w ith one another. Both conditions
37 may also be associated w ith a positive p-ANCA despite the absence of vasculitis in both conditions.
38
39
Time Spent: 3 seconds Copyright UW orld Last updated: [2/19/20151
1
2
" Item: 41 of 44
Q. ld: 4564 [
IIPMark <J
Previous
[:>
Next
jf
Lab Values
~
Notes
~
Calculator
3
4
5
6 A 64-year-old male comes to the physician's office because of increasing pain in his right groin for the past
7 several months. The pain increases w ith activity and is relieved w ith rest. He also has difficulty moving after
8 a period of rest. He denies any trauma or falls. He has no fever, w eight loss or loss of appetite. He has had
9 lumbar disk herniation in the past but denies any current back pain . He has no other active medical problems.
10 His vital signs are w ithin normal limits. He w eighs 95 kg (21 0 lb) and is 168 em (66 in) tall. Examination
11 show s pain on passive internal rotation of right hip joint. Direct pressure over the groin did not increase the
12 pain. His reflexes are 2+, and there are no sensory deficits. Muscle bulk, tone and pow er are w ithin normal
13 limits. Pulses are 2+ in both legs. W hich of the follow ing is the most likely cause of his hip pain?
14
15
A. Cutaneous nerve compression [5%]
16
17 B. Inflammation of the trochanteric bursa [17%]
18 .; C. Degenerative joint disease [56%]
19
D. Disruption of bone vasculature [ 11 %]
20
21 r... E . Referred pain from the lumbosacral area [11 %]
22
23
Explanation: User ld:
24
25 The patient described is most likely suffering from degenerative joint disease (osteoarthritis, "wear and tear"
26 arthritis). Osteoarthritis typically presents indolently in patients over 40 years of age w ith progressive anterior
?.7 hip pain exacerbated by w alking and relieved by rest. Morning stiffness or stiffness after prolonged resting is a
18 consistent feature, but in contrast to rheumatoid arthritis, the morning stiffness of osteoarthritis lasts less than
19 30 to 60 minutes. Active and passive internal and external rotation of the hip is limited on exam . Tenderness
20 to palpation and systemic symptoms are characteristically absent in osteoarthritis, and pain at rest or
21 overnight occurs only in advanced disease. The etiology is progressive mechanical destruction of the
22 articular cartilage leading to bone-on-bone friction and the formation of osteophytes. Osteoarthritis is the most
23 common joint disease. Predisposing factors are advanced age and obesity, particularly for hip and knee
24 arthritis.
~?
18 (Choice A) Cutaneous nerve compression may cause hip pain in meralgia paresthetic a, a condition w here
19 compression of the lateral femoral cutaneous nerve causes lateral hip paresthesia unaffected by motion or
20 palpation .
21
22 (Choice B) Inflammation of the trochanteric bursa (trochanteric bursitis) is caused by friction between the
23 tendons of the gluteus medius and tensor fascia lata over the greater trochanter of the femur. Pain is
24 localized over the lateral hip and is w orsened by palpation . Pain caused by pressure on the lateral hip may
25 interfere w ith sleeping in patients w ith this condition.
26
27 (Choice D) Disruption of bone vasculature may cause osteonecrosis. Osteonecrosis of the femoral head is
28 typically associated w ith chronic corticosteroid use. W hile this condition may also present w ith chronic
29 progressive anterior hip I groin pain, there is no history of chronic corticosteroid use.
30
(Choice E) Referred pain from the lumbosacral area can cause posterior hip or gluteal pain and w hen
31
impingement of the lumbar nerve roots occurs pain may radiate laterally dow n the thigh . Such patients
32
typically have no limitation of hip mobility but w ill exhibit positive findings on a straight leg raise maneuver.
33
34 Educational objective:
35 Degenerative joint disease (osteoarthritis) presents in adults over age 40 w ith indolent progressive anterior hip
36 pain w orsened by activity and relieved by rest. The hip is not tender and systemic symptoms are absent.
37 Short-lived morning stiffness may occur.
38
39
Time Spent: 30 seconds Copyright UW orld Last updated: [2119/2015]
40
- . .. .
1
2
" Item: 42 of 44
Q.ld: 2317 [
l ll''Mark <:J
Previous
t>
Next
il
Lab Values
~
Notes
~
Calculator
3
4
5
6 A 33-year-old African-American w oman presents with one w eek of painful skin lesions on her legs. She has
7 no cough, shortness of breath or bow el symptoms, and denies any recent illness or travel. She has no other
8 significant past medical history and does not take any medication. She smokes one pack of cigarettes and
9 drinks one glass of wine daily. She denies a history of sexually transmitted diseases and has been married to
10 a monogamous partner for the past eight years. Her mother w as diagnosed with ovarian cancer at age 65.
11 Her temperature is 37.2c (98.9.F), and blood pressure is 126/76 mmHg. On examination, she has multiple
12 tender pink to reddish nodules noted below the knee on the ex1ensor surface, as shown below.
13
14
15
16
17
18
19
20
21
22
23
24
25
26
?.7
18
19
20
21
22
23
24
25
26
?,7 Displayed wl1:h permission from Sprlnctr H. .lthcart Ltd.
18 OCopyrl.cht 2001 by Current Medicine
19
20
21 Which of the following is the most appropriate nex1 step in her management?
22
23 A. CT scan of the abdomen [1 0%]
24
B. HIV testing [5%]
25
26 .; C. Chest x-ray [52%]
?.7 D. Rectal swab for culture [2%]
18
E. Colonoscopy [9%]
19
20 F. Skin biopsy [ 19%]
21 G. PPD skin testing [3%]
22
23
24 Explanation: User ld:
25
26 This image demonstrates erythema nodosum (EN). EN is a condition in which painful, subcutaneous nodules
?,7 develop on the anterior surface of the low er legs. It is most common in w omen aged 15-40 years. EN tends
18 to be a relatively benign condition which heals on its own within a few w eeks. However, it can be an early
19 symptom of more serious disease processes and rapid identification of the underlying cause may prevent
20 disease-specific morbidity. Recent streptococcal infection is believed to be the most common cause of EN,
21 while other clinically important causes include sarcoidosis, TB, histoplasmosis, and inflammatory bow el
disease. Sarcoidosis is most common in African-American w omen. Therefore, the most appropriate nex1
18 step in her management w ould be a chest x-ray. Bilateral hilar adenopathy or interstitial lung disease w ould
19 be consistent with the diagnosis.
20
21 (Choice A, 0, & E) Typically, EN occurs around the time of inflammatory bow el disease exacerbation. This
22 patient has no bow el symptoms and does not need a CT of the abdomen, colonoscopy, or stool cultures.
23
24 (Choice B) HIV is a rare cause of EN. HIV testing is not part of the initial w ork-up.
25
26 (Choice F) The cause of EN is most often diagnosed by history, physical exam, and symptom-guided w ork-
27 up. How ever, skin biopsy is performed in atypical cases.
28
(Choice G) The initial w ork-up of EN is symptom-guided and includes chest x-ray, PPD skin testing, and
29
Antistreptolysin-a (ASO) titer. How ever, no risk factors for TB are revealed in this patient's history, so a chest
30
x-ray is more pressing than tuberculin skin test.
31
32 Educational objective:
33 Erythema Nodosum (EN) is a condition of painful, subcutaneous, pretibial nodules. It can be a symptom of
34 more serious disease processes including sarcoidosis, TB, histoplasmosis, recent streptococcal infection,
35 and inflammatory bow el disease. The association of EN with sarcoidosis is particularly strong in young,
36 African-American w omen. Cough, arthritis, uveitis and hilar adenopathy on chest x-ray are also associated
37 with sarcoidosis.
38
39
Time Spent: 24 seconds Copyright UW orld Last updated: [2/ 12/2015]
40
1
2
" Item: 43 of 44
Q. ld: 2314 [
11 P Mark <J
Previous
[:>
Next
jf
Lab Values
~
Notes
~
Calculator
3
4
5
6 A 50-year-old man presents to your office complaining of pain and sw elling of the right knee. He bumped his
7 right knee into a pole 2 days ago w hile w orking. He had one episode of similar pain two years ago that
8 resolved w ith over-the-counter analgesics. He denies any illicit drug use. He has no other medical problems
9 and does not take any medications. His temperature is 37.2c (98.9.F), and blood pressure is 126/76
10 mmHg. Examination reveals sw elling, w armth and decreased range of motion of the right knee. All other
11 joints are w ithin normal limits. Synovial fluid analysis show s positive birefringent crystals and negative Gram
12 stain. These crystals are most likely composed of?
13
14
15 A. Hydroxyapatite [3%]
16 B. Monosodium urate [11 %]
17 .; C. Calcium pyrophosphate [79%]
18
19 D. Calcium oxalate [6%]
20 E . Ammonium phosphate [1 %]
21
22
23 Explanation: User ld:
24
This patient's history and synovial fluid analysis are suggestive of pseudogout, the acute form of calcium
25
pyrophosphate dihydrate (CPPD) crystal disease. Attacks of pseudogout often occur in the setting of trauma,
26
surgery, or medical illness. This patient's knee trauma w as the likely precipitant of his condition . Patients w ith
?.7
pseudogout present w ith acute pain, swelling, redness, and limited motion of the involved joint(s), w ith the
18
knee being most commonly affected (>50% of cases). Fever, leukocy1osis w ith a left shift, and
19
chondrocalcinosis (calcified articular cartilage) may also occur in pseudogout. W hile this patient's history is
20
consistent w ith pseudogout, gout and septic arthritis may present similarly. Therefore, the diagnosis cannot
18 be made based on history and examination alone. Synovial fluid analysis is critical for distinguishing betw een
19 these conditions. The identification of rhomboid, positively birefringent crystals on synovial fluid analysis is
20 diagnostic of pseudogout.
21
22 (Choice A} Hydroxyapatite is the complexed form of calcium found in teeth and bones. How ever, it can also
23 exist in the cartilage of patients w ith degenerative osteoarthritis and may be isolated from synovial fluid of such
24 patients.
25
26 (Choice B) Monosodium urate crystal deposition is seen in gout. These crystals are needle-shaped and
27 negatively birefringent on synovial fluid analysis.
28
29 (Choice D) Calcium oxalate is the most common crystal found in renal calculi. Calcium oxalate crystals are
30 not isolated from synovial fluid.
31
(Choice E) Struvite (magnesium ammonium phosphate) may be found in the renal calculi of patients w ith
32
urinary tract infection caused by urease-producing organisms (Klebsiella, Proteus).
33
34 Educational objective:
35 Pseudogout cannot be reliably distinguished from gout and septic arthritis based on history and physical
36 alone. It is diagnosed by the presence of rhomboid, positively birefringent crystals on synovial fluid analysis,
37 and radiographic evidence of chondrocalcinosis.
38
39
Time Spent: 4 seconds Copyright UW orld Last updated: [2/19/2015]
40
1
2
" Item: 44 of 44
Q. ld: 3516 [
IIPMark <J
Previous
[:>
Next
jf
Lab Values
~
Notes
~
Calculator
3
4
5
6 A 68-year-old Caucasian male complains of right-sided leg pain over the last year. The pain is poorly
7 controlled w ith over-the-counter pain medications. He also complains of frequent headaches. His past
8 medical history is significant for hypertension and recent hearing loss. An X-ray of the skull show s areas of
9 bone resorption and sclerosis. W hich of the follow ing are the most likely findings on laboratory w ork-up?
10 . Serum Alkaline Urinary
11 Serum calcrumphosphate phosphatase hydroxyproline
12
13
14 A. Normal Normal Normal High [2%]
15
16 B. Normal Normal High High [58%]
17
18 C. High Low High Normal [33%]
19
20 D. High Normal Normal Normal [3%]
21
22 E. Low High Normal Normal [2%]
23
24
?F Explanation: User ld:
18
19 The patient described is most likely suffering from osteitis deformans (Paget disease of bone). Paget disease
20 is a disorder of accelerated and disordered osteoclastic bone resorption of uncertain etiology. Increased bone
21 resorption in this disease is answ ered by disorganized formation of lamellar or w oven bone in affected sites
22 that is inherently w eaker than normally formed bone. The most commonly involved sites are the femur, axial
23 skeleton, and skull. Involvement of the w eight-bearing bones of the low er extremities causes pain and bow ing
24 of the bone. Hypertrophy of the skull can compress the vestibulocochlear nerve as it passes through the
25 internal acoustic meatus resulting in deafness. Laboratory abnormalities seen in Paget disease of bone
26 include normal calcium and phosphate levels and an elevated alkaline phosphatase level. Urinary markers of
27 bone degradation such as hydroxyproline, deoxypyridinoline, N-telopeptide and C-telopeptide are elevated as
28 w ell.
29
30 (Choice C) Hyperparathyroidism is associated w ith hypercalcemia, hypophosphatemia and an increased
31 alkaline phosphatase.
32
(Choice D) Hypercalcemia in the setting of normal phosphate and alkaline phosphatase levels may be
33
observed in the milk-alkali syndrome.
34
35 Educational objective:
36 Paget disease of bone (osteitis deformans) is associated w ith normal serum calcium and phosphate levels
37 and increased alkaline phosphatase and urinary hydroxyproline levels.
38
39
Time Spent: 7 seconds Copyright UW orld Last updated: [2/19/2015]
40
- + - : '
UWo rld Step 2 QBank
I
Welcome Create I I
t Previous Test Cumulative Performance I Search I Utilities I Help I
List of Tests J Review Test ] I
Avg. Score: 66%
I Get Question list I Sort by: IQuestion Sequence vi [&}
Your Score: 0% Display : @ All 0 Correct 0 Incorrect 0 Omitted 0 Marke<l
,.,
MQ# Main Division Sub Division Topic

! 1-13170) Medicine Rheumatology/Orthopedics & Sp... Rotator Cuff Tear

! 2 -14581 ) Medicine Rheumatology/Orthopedics & Sp... Psoriatic Arthritis

! 3 -14743) Medicine Rheumatology/Orthopedics & Sp... Neuropathic Joints (Charcot)

! 4- [4587] Medicine Rheumatology/Orthopedics & Sp... SLE And Joints

! 5-14110] Medicine Rheumatology/Orthopedics & Sp... DJD

! 6- [3574) Medicine Rheumatology/Orthopedics & Sp... Suoacromial Bursitis

! 7 -14582) Medicine Rheumatology/Orthopedics & Sp... Pseudogout

I 8 -1331 7] Medicine Rheumatology/Orthopedics & Sp... Polymyalgia Rheumatica

! 9 -14574) Medicine Rheumatology/Orthopedics & Sp... RA And Cervical Spine

! 10 - 19906] Medicine Rheumatology/Orthopedics & Sp... Raynaud Treatment

! 11 - 14590) Medicine Rheumatology/Orthopedics & Sp... Disseminated Gonococcemia

! 12 - 13575) Medicine Rheumatology/Orthopedics & Sp. . Lateral Epicondylitis (Tennis Elb...

! 13 - 13164) Medicine Rheumatology/Orthopedics & Sp. . Temporal Arteritis And Vision Loss

! 14- 135 15] Medicine Rheumatology/Orthopedics & Sp... CREST Syndrome

! 15- 13176] Medicine Rheumatology/Orthopedics & Sp... Sarcoidosis

I 16- [4572) Medicine Rheumatology/Orthopedics & Sp. . Rheumatoid Arthritis And Osteop...

! 17- 13311 ) Medicine Rheumatology/Orthopedics & Sp... Whipple Disease

! 18- 13153) Medicine Rheumatology/Orthopedics & Sp... Neisseria Gonorrhoeae

! 19 - 14580) Medicine Rheumatology/Orthopedics & Sp... Fioromyalgia Diagnosis

! 20 - 131 48) Medicine Rheumatology/Orthopedics & Sp... Osteoarthritis

! 2 1 - 14078) Medicine Rheumatology/Orthopedics & Sp... Lumoosacral Strain

! 22 - 13157] Medicine Rheumatology/Orthopedics & Sp... Spinal Stenosis

! 23 - 13304) Medicine Rheumatology/Orthopedics & Sp... Pager s Disease

I 24- [4565) Medicine Rheumatology/Orthopedics & Sp... Osteonecrosis, Steroid Induced

! 25 - 13155) Medicine Rheumatology/Orthopedics & Sp... Osteoarthritis

! 26 - 14460) Medicine Rheumatology/Orthopedics & Sp... Temporal Arteritis

! 27- 13167] Medicine Rheumatology/Orthopedics & Sp... Ankylosing Spondylitis

! 28 - 18895) Medicine Rheumatology/Orthopedics & Sp... ACL Injury

! 29- 13165] Medicine Rheumatology/Orthopedics & Sp... Ankylosing Spondylitis

! 30 - 13150) Medicine Rheumatology/Orthopedics & Sp... Acute Gouty Arthritis

! 31 - 14585] Medicine Rheumatology/Orthopedics & Sp... AA Amyloidosis

I 32- [3898) Medicine Rheumatology/Orthopedics & Sp... SLE Nephritis

! 33 -13203) Medicine Rheumatology/Orthopedics & Sp... Carpal Tunnel Syndrome

! 34 -13209) Medicine Rheumatology/Orthopedics & Sp... Polymyositis

! 35 - 14264) Medicine Rheumatology/Orthopedics & Sp... Acute Lower Lumoar Back Pain

! 36 - 14369) Medicine Rheumatology/Orthopedics & Sp... Lumoar Spinal Stenosis

! 37 - 13166) Medicine Rheumatology/Orthopedics & Sp... Ankylosing Spondylitis

! 38 - 13318) Medicine Rheumatology/Orthopedics & Sp... Rheumatoid Arthritis

! 39 - 14205] Medicine Rheumatology/Orthopedics & Sp... DJD

I 40- [3312) Medicine Rheumatology/Orthopedics & Sp... lnftammatory Bowel Disease

! 41 - 14564) Medicine Rheumatology/Orthopedics & Sp... Osteoarthritis- Hip Pain

! 42 - 12317] Medicine Rheumatology/Orthopedics & Sp... Sarcoidosis

! 43- [23 14) Medicine Rheumatology/Orthopedics & Sp... Pseudogout

! 44 - 13516) Medicine Rheumatology/Orthopedics & Sp... Paget Disease Of Bone

...
Cop)Tig ht USM LEWOfld, LLC

S-ar putea să vă placă și